Sie sind auf Seite 1von 321

The connaissance

Grand Scholastic
Tome
Part II
Compiled by Jamesdrrr

2020 Edition
This document is not for sale.

The Connaissance Grand Scholastic Tome


Compiled by
Jamesdrrr

Contents
Topic Subtopics Topic Code Page Number
M A T H E M A T I C S
Sets  Sets Math-Sets 6
 Types of Sets
 Numerical Sets
 Set Operations
 Set Relationships
Types of Numbers  Types of Numbers Math-Num 8
and their Properties  The Number Line
 Interval Notation
 Properties of Numbers
Fractions, Decimals,  Fractions, Decimals, and Math-FracDec 15
and Percentage Percentage
 Operations on Fractions
 Operations on Decimals
 Operations on Percentages
 Conversion
Scientific Notation  Scientific Notation Math-SciNot 21
Unit Conversion  Unit Conversion Math-Unit 21
 Conversion of Distance Units
 Conversion of Temperature Units
 Conversion of Volume and
Capacity Units
 Conversion of Mass/Weight Units
 Conversion of Area Units

1
This document is not for sale.

 Conversion of Miscellaneous
Units
Phrase Conversion  Phrase Conversion Math-Phrase 34
Exponents and  Exponents and Radicals Math-ExRad 35
Radicals  Laws of Exponents and Radicals
Algebra  Algebra Math-Algebra 36
 Terminologies
 Functions and their
Classifications
 Rules and Properties
 Operations
 Revisiting Complex Fractions
An Introduction to  Geometry Math-Geometry 43
Geometry  Terminologies
Rectangular  Rectangular Coordinate System Math-RecCoor 51
Coordinate System
Basic and Analytic  Diagonals Math-AnaGeo 53
Geometry  Angles
 Triangles
 Circles
 Quadrilaterals
 Parabola
 Ellipse
 Hyperbola
 Solid Geometry
Factoring  Factoring Math-Factor 101
 Methods of Factoring
Hypotheses and  Hypotheses and Arguments Math-HypoArg 103
Arguments  Definitions
 Theorems
 Properties and Postulates
Linear Equations and  Linear Equation Math-Linear 115
Inequalities  Linear Inequality
 Linear Function
Quadratic Equations  Quadratic Equation and Function Math-Quad 120
and Inequalities  Quadratic Inequality
Trigonometry  Trigonometry Math-Trig 130
 Pythagorean Theorem
 Terminologies and Formulae
 Unit Circle

2
This document is not for sale.

Patterns and  Patterns and Sequences Math-PatSeq 141


Sequences  Number Patterns
 Arithmetic Sequence
 Geometric Sequence
 Harmonic Sequence
 Fibonacci Sequence
Pascal’s Triangles  Pascal’s Triangle Math-Pascal 146
Division of  Long Division Math-DivPol 146
Polynomials  Synthetic Division
Permutation and  Permutation Math-PermCom 153
Combination  Combination
Measures of Position  Measures of Position Math-Position 160
 Decile
 Quartile
 Percentile
Functions  Functions Math-Function 169
 Revisiting Functions
 Combination of Functions
 Piece-Wise Functions
 Inverse Functions
 Polynomial Functions
 Rational Functions
 Radical Functions
 Exponential Functions
 Logarithmic Functions
 Trigonometric Functions
 Composite Functions
Business  Business Mathematics Math-BusMath 200
Mathematics  Terminologies
 Simple Interest
 Compound Interest
Logic and the Truth  Logic and the Truth Table Math-Logic 207
Table
Mathematical  Mathematical Induction Math-MatInd 211
Induction
Statistics  Statistics Math-Stats 211
 Measures of Central Tendencies
 Probability
 Combinatories
 Set Theory

3
This document is not for sale.

 Mean, Variance, and Standard


Deviation
 Formulating Hypothesis
Sigma Notation  Sigma Notation Math-Sigma 228
An Introduction to  An Introduction to Calculus Math-Calc 229
Calculus  Limits of a Function
 Continuity
Differentiation  Differentiation Math-Diff 242
 Maxima and Minima
Integration  Integration Math-Integ 266
Formula Summary  Formula Summary Math-Formula 292
A P P E N D I X
References 316
Additional Information 320

4
This document is not for sale.

Mathematics
This subject focuses on numbers, logic, and the operations that can
be done with them. It deals with fields such as Algebra, Calculus,
Trigonometry, Statistics, and more.

HS and SHS subjects referenced:


HS Mathematics 7, 8, 9, & 10
SHS General Mathematics
SHS Statistics and Probability
SHS Pre-Calculus
SHS Basic Calculus

5
This document is not for sale.

Sets
In Mathematics, a set is defined as a collection of well defined and distinct objects that is considered as an
object in its own right. Sets have a specific notation in mathematics:

{3, 6, 91,…}

The set is enclosed in curly bracket or braces { }, each element or member (a single component of the set) is
separated by a comma. Sets, especially infinite sets, typically has an ellipsis/ellipses (…) as the last element to
denote infinity. Ellipses can also be used to denote a skip or an omission from the set.

{3, 6, 91,… 100}

Sets are typically named using capital letters.

A = {3, 6, 91,… 100}, this is set A

When we say an element a is in a set A, we use the symbol to show it, and if something is not in a set use .

A = {3, 6, 91, 100}, 6 A, but 8 A.

Set builder notation can also be used so we can merely describe the elements instead of listing them down,
typically used in universal sets.

{x|x>0} is the set of all x’s, such that x is greater than 0. Basically, this set contains any number greater than
zero.

Types of Sets

Infinite Sets: Sets that have no ending or have no defined number of elements.

Ex. A set of all the stars in the universe. {Sun, Proxima Centauri, …} since the exact number of stars in the
universe is undefined, therefore, infinite.

Finite Sets: Sets that have a defined number of elements.

Ex. A set of all the letters in the English alphabet. {a, b, c, …, x, y, z} since no matter what, there will always
be 26 letters in the English alphabet from a to z.

Null Sets: Sets that contain nothing. Denoted by empty curly brackets or slashed zero (∅). These sets are
subsets of lall sets including itself.

Universal Sets: Sets that contain everything that is relevant to the situation.

Numerical Sets

Numbers can also be defined using sets.

6
This document is not for sale.

Ex.

Set of even numbers: {..., −4, −2, 0, 2, 4, ...}

Set of odd numbers: {..., −3, −1, 1, 3, ...}

Set of prime numbers: {2, 3, 5, 7, 11, 13, 17, ...}

Positive multiples of 3 that are less than 10: {3, 6, 9}

Sets can also be defined based on the properties of its numerical elements.

Ex.

{x|x>0} means "the set of all x's, such that x is greater than 0"

Numbers can also be defined in universal sets.

Ex.

In number Theory, the universal set is all integers since number theory is the study of integers.

In Calculus, the universal set is real numbers.

In complex analysis, the universal set is complex numbers.

Set Operations

7
This document is not for sale.

Union: The combination of sets. A set that contains all the elements from combined sets. A∪B={x∣x∈A∨x∈B}.

Ex. {1,2,3,4}∪{3,4,5,6}={1,2,3,4,5,6}

Theorem: |A∪B|≥|A| and |A∪B|≥|B|

Intersection: A set that contains elements that are present in the both of two different sets.
A∩B={x∣x∈A∧x∈B}.

Ex. {1,2,3,4}∩{3,4,5,6}={3,4}

Theorem: |A∩B|≤|A| and |A∩B|≤|B|

Difference: The set of values in one set that is not in the other. A−B={x∣x∈A and x∉B}.

Ex. {1,2,3,4}−{3,4,5,6}={1,2}

Theorem: For any sets, |A−B|≤|A|

Complement: The set of all values not in a set. Denoted as X’ meaning the complement of set X’.
S’={x∣x∉S}=U−S.

Ex. U={1,2,3,4,5,6,7,8}, A={1,2,3,4}, A’={5,6,7,8}

Theorem: For any sets, S∩S’= ∅

Set Relationships

Equality: When sets have precisely the same members.

Ex. A={1,2,3,4}, B={3,2,1,4}, Set A and B are equal. A=B.

Subset: Set whose elements are all members of another set. A is a subset of B if and only if every element of A
is in B.

Ex. A={1,2,3,4,5,6,7,8}, D={1,2,3,4}, D⊆A, Y={4}, Y⊆A

Proper Subset: Set whose elements are members of another set, but not all. A is a proper subset of B if and
only if every element of A is also in B, and there exists at least one element in B that is not in A.

Ex. A={1,2,3,4,5,6,7,8}, D={1,2,3,4,5,6,7}, Set D is a Proper Subset of A or D⊂A

Types of Numbers and their Properties


In Mathematics, there are many different types of numbers.

8
This document is not for sale.

Natural Numbers: Also known as positive integers, or counting numbers. N={1,2,3,4,5,…}

Whole Numbers: Natural numbers with zero. W={0,1,2,3,4,5,…}

Integers: All whole numbers including the negative numbers. Z={…,-2,-1,0,1,2,…}

Rational Numbers: Fractions in which the numerators and denominators are integers. The denominator cannot
be zero, but the numerator can. Q={…,-1,-1/2,-1/4,0,1/4,1/2,1,…}

Irrational Numbers: All real numbers that are not Rational Numbers. Numbers that can’t be simply written as
a fraction/decimal. Q’={10/3,.3333333333333,…}

Real Numbers: Includes all the numbers that can be written as decimals, whether it be fractions, integers, or
radicals. R={,5,-0.43534354,…}

Imaginary Numbers: A complex number that can be denoted as a real number multiplied by the imaginary unit
i. In mathematics, the unit i2=-1. i={-1,2i,…}

We can also use these different Types of Numbers in set Notations

Ex. {x R|x3} is read as the set of all x’s that are a member of the Real Numbers, such that x is greater than or
equal to 3. This set basically contains all real numbers that are greater than or equal to three which can look like
this {3, , 3.5, 4, 9/2,…}

The Number Line

The Number line can also be used in set notation o in inequality notations.

9
This document is not for sale.

Interval Notation

10
This document is not for sale.

11
This document is not for sale.

Properties of Numbers

12
This document is not for sale.

13
This document is not for sale.

14
This document is not for sale.

Fractions, Decimals, and Percentage


Whole numbers can be divided into smaller subunits, which can be represented using fractions, decimals, or
percentages.

Operations on Fractions

Improper Fraction to Mixed Numbers

1. Divide the numerator and the denominator.

2. Use the quotient as the whole number.

3. Use the remainder as the numerator.

4. Retain the denominator.

15
This document is not for sale.

Mixed Numbers to Improper Fraction

1. Multiply the denominator to the whole number.

2. Add the product of step 1 to the original numerator.

3. Use the sum of step 2 as the new numerator.

4. Retain the old denominator.

Addition and Subtraction of Fractions

1. Find the Least common denominator.

2. Write both original fractions as equivalent fractions with the least common denominator.

3. Now that both fractions have the same denominator, add or subtract the numerators.

4. Use the least Common Denominator as the denominator of your final answer.

16
This document is not for sale.

Multiplication of Fractions

1. Multiply the numerator to the numerator.

2. Multiply the denominator to the denominator

Division of Fractions

1. Get the reciprocal of the divisor.

2. Apply the same steps for the Multiplication of fractions.

17
This document is not for sale.

Operations on Decimals

Addition and Subtraction of Decimals

1. Line Up the Decimal Points.

2. Follow the typical steps for adding or subtracting any number.

18
This document is not for sale.

Multiplication of Decimals

1. Do not line up the decimal points.

2. Follow the typical rules of Multiplication.

3. Count the total number of places to the right of the decimal point in both factors, use that number as the
number of digits to the right of the decimal point.

Division of Decimals

1. Estimate the answer by rounding. You'll use this estimate to check your answer later.

2. If the divisor is not a whole number, then move the decimal place n places to the right to make it a whole
number. Then move the decimal place in the dividend the same number of places to the right (adding some
extra zeros if necessary.)

3. Divide as usual. If the divisor doesn't go in evenly, add zeros to the right of the dividend and keep dividing
until you get a 0 remainder, or until a repeating pattern shows up.

4. Put the decimal point in the quotient directly above where the decimal point now is in the dividend.

5. Check your answer against your estimate to see if it's reasonable.

19
This document is not for sale.

Operations on Percentages

Percentages are basically a different notation of decimals.

Operations on Percentages typically follow the same rules as typical addition, subtraction, multiplication, or
division. You can also convert the percentages to their decimal version and proceed with the operations there.

Conversion

Fraction to Decimal: Divide the denominator and numerator, using the numerator as the dividend, and the
denominator as the divisor.

Ex. ¼ = 1  4 = 0.25

Fraction to Percent: Multiply the fraction by 100 then add a percent sign.

Ex. ¼ = ¼ x 100 = 100/4 = 100  4 = 25 = 25%

Decimal to Fraction: Count the decimal places. If there is one decimal place, use 10 as the denominator, if
there are two, use 100, if there are three, use 1000, and so on. Then, reduce to simplest form.

Ex. 0.25 = 25/100 = ¼

Decimal to Percent: Move the decimal point two places to the right then add a percent sign.

Ex. 0.25 = 25%

Percent to Decimal: Move the decimal two places to the left then remove the percent sign.

20
This document is not for sale.

Ex. 25% = 0.25

Percent to Fraction: Remove the percent sign, use 100 as the denominator, then reduce.

Ex. 25% - 25/100 = ¼

Scientific Notation
Scientific Notation is a method generally used in scientific fields of writing very large or very small numbers I a
short and concise manner.

1. Get the factors of the number, with one factor typically being a multiple of 10.

2. Get the exponent form of the factor that is a multiple of 10.

Ex.

700 = 7 x 100 = 7 x 102

4,900,000,000 = 4.9 x 1,000,000,000 = 4.9 x 109

5326.6 = 5.3266 x 1000 = 5.3266 x 103

Scientific Notation with small numbers typically work in a different way. You still have to factor, but now
instead of multiples of 10, you go the other way round. The general tip is that if a number is 10 or greater, the
decimal point moves to the left, and the power of 10 is positive; when the number is smaller than 1, the decimal
point has to move to the right so the power of 10 is negative.

Ex.

0.0055 = 5.5 x 0.001 = 5.5 x 10-3

3.2 = 3.2 x 1 = 3.2 x 100

Unit Conversion
Due to some strange reason, the world still uses different units to measure different things instead of using one
standard worldwide unit, therefore, The Conversion of these units is important.

The Metric System has an organized system of prefixes attached to each unit.

giga- 109

21
This document is not for sale.

(G-)
mega- 106
(M-)
kilo- 103
(k-)
hector- 102
(h-)
deka- 101
(da-, D-)
gram (g) 1
meter (m)
liter (L)
deci- 10-1
(d-)
cent- 10-2
(c-)
milli- 10-3
(m-)
micro- 10-6
(-)
nano- 10-9
(n-)

Conversion of Distance Units

English to Metric

22
This document is not for sale.

Symbol To convert from Multiply by To determine Symbol

IN inch 25.4 millimeters mm

FT feet 0.3048 meters m

YD yards 0.9144 meters m

MI miles 1.609344 kilometers km

Metric to Metric

23
This document is not for sale.

English to English

Conversion of Temperature Units

English to Metric

Metric to English

24
This document is not for sale.

Celsius to Kelvin

Conversion of Volume and Capacity Units

English to Metric

25
This document is not for sale.

Symbol To convert from Multiply by To determine Symbol

CI Cubic Inches 16.387064 cubic centimeters cm2

CF Cubic Feet 0.0283168 Cubic meters m2

CY Cubic Yards 0.764555 Cubic meters m2

GAL Gallons 3.78541 liters L

OZ Fluid Ounces 0.0295735 liters L

MBM Thousand Feet Board 2.35974 Cubic meters m2

Metric to Metric

1 cubic centimeter = 1 milliliter


1 cubic decimeter = 1 Liter

26
This document is not for sale.

English to English

27
This document is not for sale.

Conversion of Mass/Weight Units

English to Metric

Symbol To convert from Multiply by To determine Symbol

LB Pounds 0.4535924 Kilograms kg

TON Short Tons 0.9071848 Metric Tons t

Metric to Metric

28
This document is not for sale.

English to English

Conversion of Area Units

English to Metric

Symbol To convert from Multiply by To determine Symbol

SI Square Inches 645.16 Square centimeters mm2

SF Square Feet 0.09290304 Square meters m2

SY Square Yards 0.83612736 Square meters m2

A Acres 0.4046856 Hectares ha

MI2 Square Miles 2.59 Square Kilometers km2

Metric to Metric

1 ha = 0.01 km2 or 1 hectometer

29
This document is not for sale.

Conversion of Miscellaneous Units

30
This document is not for sale.

31
This document is not for sale.

32
This document is not for sale.

33
This document is not for sale.

Phrase Conversion
Mathematical Terms can be expressed either through numerical form or word form, so it is important to know
how to translate both expressions to the other.

Verbal Phrase Variable Expression

y more than x x+y

y greater than x

y added to x

x plus y

sum of x and y

y fewer than x x-y

y less than x

The difference of x and y

y less x y-x

34
This document is not for sale.

a number n

an unknown number

The product of x and a number xn or xn

The quotient of x and a number x/n or xn

Twice of a number 2n

Half of a number ½n

Square root of a number n

A number raised to two n2

Exponents and Radicals


Exponentiation determines how much times a number is to be multiplied to itself. Radicals or the
square/cube/nth root is the opposite application of exponents; a sort of “undo”.

Laws of Exponents and Radicals

The following are the Laws of Exponents where a is the base number, and n and m are the exponents

Product Rule: anam=an+m and anbn=(ab)n

Quotient Rule: an/am=an-m and an/bn=(a/b)n

35
This document is not for sale.

Power Rule: (bn)m=bnm and bn^m=b(n^m) and m(bn)=bn/m and b1/n = nb

Negative Exponents Rule: b-n=1/bn

Zero Rule: b0=1 and 0n=0 when n>0

One Rule: b1=b and 1n=1

Minus One Rule: (-1)n = 1 when n is even, and -1 when n is odd.

Derivative Rule: (xn)’ = nxn-1

Integral Rule: xndx=xn+1/(n+1)+c

The following are the Laws of Radicals where n is the exponent and the index, x is the radicand, and r is the
base number. a, b, and c are numbers.

When Adding an Exponent to a Radical: if nx=r then (nx)n = rn, thus rn = x

Adding and Subtracting Radicals: ab + cb = (a+c)b and ab - cb = (a-c)b

Multiplying Radicals: ab = ab

Simplifying Radicals: ax2 = a and ax3 = xa and 3x5 = (x)3x2 and 3x6 = x2 and so on

Radicals within Radicals: mnx = mnx

Algebra
Algebra is a broad branch of Mathematics that deals with variables, or symbols that stand for undefined
numbers, and the rules that govern these components. It is one of the main branches of mathematics that sews
together its other fields, such as geometry, number theory, arithmetic, calculus, geometrical analysis, and more.
Algebra is also known as the study of generalized arithmetic.

Terminologies

Muhammad ibn Musa al-Khwarizmi: The Father of Geometry.

Al-Jabr: Al-Khwarizmi’s book from which the name Algebra is defined.

Arithmetic: The Basic Study of Numbers and traditional operations such as addition, subtraction,
multiplication, and division.

Absolute Value: The value of the original distance of the number fro the center of the number line, regardless
of the sign. Represented using |x|, where x is a number.

36
This document is not for sale.

Variable: A placeholder for an undefined number, usually denoted using letters.

Constant: A term that is a fixed number with a defined value.

Coefficient: A fixed number.

Term: Usually composed of a constant, variable, or coefficient that are the main components of an expression.
Separated from other terms through the operations of addition and subtraction.

Expression: A mathematical statement that consists of one or more constants, variables, and operations.

Equation: A mathematical statement that contains an equal sign.

Monomial: A mathematical expression with one term.

Binomial: A mathematical expression with two terms.

Trinomial: A mathematical expression with three terms.

Polynomial: The general combination of terms and operations.

Inequality: A Mathematical statement that contains an inequality symbol. Such as less than (<), greater than
(>), less than or equal to (≤), greater than or equal to (≥), and not equal to (≠).

Degree: The greatest value of the exponent in an expression or equation. When the exponent is not written, it is
automatically 1.

Function: An equation that only has one answer for the value of y for every x. Functions can typically be
graphed and are on of the foundations of analytical geometry,

37
This document is not for sale.

Functions and their Classifications

There are certain steps/methods/rules in order for us to determine what expression is a function, and what isn’t.
Since functions must only have one value for y for every value of x, we could:

1. Examine Ordered Pairs

2. Solve for Y

38
This document is not for sale.

3. Do the vertical line test

4. Use an input-output chart

The following functions can be classified depending on its degree.

Constant: Has a degree of 0. P(x) = a or ax0

39
This document is not for sale.

Linear: Has a degree of 1. P(x) = ax + b

Quadratic: Has a degree of 2. P(x) = ax2+bx+c

Cubic: Has a degree of 3. P(x) = ax3+bx2+cx+d

And so on.

Rules and Properties

Besides the common properties of numbers, algebra has additional properties.

 Remember that when multiplying or dividing same signs, the answer shall always have a positive sign.

 When multiplying different signs, the answer shall always have a negative sign.

 PEMDAS Order of Operations shall always be followed from left to right. (Parenthesis, Exponent,
Multiplication/Division, Addition/Subtraction)

 Always simplify the expression BEFORE solving.

Operations

Addition and Subtraction: Group all the similar terms, then combine all the like terms and the same variables.
Only variables with the same exponent can be added or subtracted, which in turn shall increase or decrease their
accompanying coefficient.

40
This document is not for sale.

Ex.

x + 5 + 3x + 5y – 2 + 5

x + 3x + 5y – 2 + 5

4x + 5y + 3

Ex.

5x + x2 – 3x + 5

x2 + 5x -3x + 5

x2 + 2x + 5

Multiplication: Distribute the first term to all the terms of the second factor, then distribute the second term to
all the terms of the second factor, then the third, then the fourth, and so on. Remember to add similar terms.

Ex.

x(x+1)

x(x) + x(1)

x2+x

Ex.

(x-2)2

(x-2)(x-2)

x(x)+x(-2)-2(x)-2(-2)

x2-2x-2x+4

x2-4x+4

This forms a perfect square trinomial, which is a trinomial that happens to be the result of a binomial that is
multiplied to itself.

Division: Division of Algebraic expressions is basically the simplification of terms.

Ex.

30a3b2  5a2b3

30 aaa bb  5 aa bbb

41
This document is not for sale.

6a  b

6a/b

Revisiting Complex Fractions

Complex fractions are fractions within fractions or fractions that have operations between fractions within them,
and need to be simplified before we can do anything with them. Remember that fractions express the process of
division, so simplifying complex fractions mostly involve the division of fractions.

42
This document is not for sale.

An Introduction of Geometry
Geometry is a Branch of Mathematics dealing with the study of shapes, figures, its properties, and the proof
governing everything about the properties of said shapes.

Terminologies

Euclid: The father of geometry.

Point: An element or a speck in a space. A zero dimensional figure.

Line: A straight one dimensional figure made of points that extends infinitely on both sides.

Line Segment: Similar to a line, except it has defined endpoints on both sides.

Ray: Similar to a line, except it has a defined endpoint on only one side.

Plane: A flat two dimensional surface that extends infinitely far. Made up of countless points, lines, etc.

43
This document is not for sale.

Angle: A figure formed by two rays meeting at a common endpoint. Denoted using the symbol ∠

 Acute – An angle that has less than 90

 Right – An angle that has exactly 90. Denoted using the symbol ∟

 Obtuse – An angle that has more than 90 but less than 180

 Straight – An angle that has exactly 180

 Reflex – An angle that has more than 180 but less than 360

 Full Rotation – An angle that has exactly 360

 Complementary – Two angles that, when combined with each other, have a total degree of 90

 Supplementary – Two angles that, when combined with each other, have a total degree of 180

44
This document is not for sale.

Degree: Measurement between the two rays or arms of the angle. Denoted using the symbol °

Vertex: The point where the two rays of the angle meet.

Parallel: Lines on the same plane that do not meet. Denoted using the symbol ∥

Intersecting: When two or more lines meet on one point.

Perpendicular: Intersecting lines that form a right angle. Denoted using the symbol ⊥

Polygon: A closed plane two dimensional figure made up of lines/segments and angles.

Triangle: A Polygon with three sides and three angles. Denoted using the symbol Δ

 Equilateral – All three sides have the same measurements

 Isosceles – Two sides have the same measurements

 Scalene – No sides have the same measurements

 Equiangular – All angles have the same measurements

 Acute – All three angles are less than is 90 degrees in measurement

 Right – Has one angle that is 90 degrees in measurement

 Obtuse - Has one angle that is greater than 90 degrees in measurement

45
This document is not for sale.

Quadrilateral: A polygon with four sides and four angles.

 Rectangle – Has four right angles, wherein the sides that are parallel or opposite to each other have the
same measurement.

 Square – Has four right angles, and all four sides are equal in measurement.

 Rhombus – Opposite angles have the same degree and all four sides are equal in measurement.

 Parallelogram – Opposite sides are parallel to each other.

 Trapezoid – One pair of opposite sides are parallel.

 Kite – Adjacent sides are equal in measurement.

46
This document is not for sale.

Pentagon: A polygon with five sides and five angles.

Hexagon: A polygon with six sides and six angles.

Heptagon: A polygon with seven sides and seven angles.

Octagon: A polygon with eight sides and eight angles.

Nonagon: A polygon with nine sides and nine angles.

Decagon: A polygon with ten sides and ten angles.

Undecagon: A polygon with eleven sides and eleven angles.

Dodecagon: A polygon with twelve sides and twelve angles.

N-gon: A polygon with an undefined or unknown amount of sides and angles. E.g. an 8-gon is also known as an
octagon.

Circle: A shape made from points that are all equidistant from a central point.

Congruence: Two different entities that have the exact same characteristics. Different to equality, since
equality suggests that the two entities are the same. Uses the symbol ≅.

Perimeter: The path that encompasses a two dimensional shape.

Circumference: The distance around the circle. The perimeter of the circle.

Area: Space occupied by a flat shape.

Volume: Space occupied by a 3 dimensional object.

Solid: A three dimensional Figure.

Bisect: To divide something into two equal parts.

47
This document is not for sale.

Transversal Line: A line that passes through two lines in the same plane. Used to establish whether the other
two lines are parallel.

48
This document is not for sale.

The following Geometric figures can be named accordingly.

49
This document is not for sale.

50
This document is not for sale.

Rectangular Coordinate System


Also known as the Cartesian Plane, the Rectangular coordinate system is a two dimensional plane consisting of
an x-axis going horizontally, and a y-axis going vertically. Both axes are perpendicular to each other and are
used for plotting and graphing various geometric figures. It was made by Rene Descartes.

The origin is at point (0,0), everything left of the origin has negative x value, everything right has a positive x
value. Everything above the origin has a positive y value, everything below has a negative y value. The plane is
divided into four quadrants.

51
This document is not for sale.

Ordered Pair: Contains the coordinates of a point in the form (x,y).

Domain: All input values shown on the x-axis

Range: All the possible output values shown on the y-axis

Abscissa: Refers to the x-axis, or the x value of the ordered pair/point.

Ordinate: Refers to the y-axis, or the y value of the ordered pair/point.

X Intercept: Where the line/graph crosses the x-axis.

52
This document is not for sale.

Y Intercept: Where the line/graph crosses the y-axis.

Basic and Analytic Geometry


Diagonals

Diagonals are lines that join the non-adjacent corners or angles of a polygon. Diagonals cannot link two
adjacent angles since the lines that link them are the sides of the polygon itself. Diagonals can also join the non-
adjacent corners of solids.

An n-sided polygon has n(n-3)/2 number of diagonals. E.g. a quadrilateral has 2 diagonals, and an octagon has
20.

When looking for the measurement of a diagonal in a quadrilateral with a right angle, it can be observed that a
right triangle is formed, thus, the Pythagorean theorem can be utilized, with a and b as the sides of the
quadrilateral and c as the diagonal. a 2+b2=c2

In squares, the formula for the length of a diagonal can be simplified to s2+s2 which can be simplified into s2.

In rectangles, the formula is diagonals=w2+l2

In trapezoids, the Pythagorean theorem is used, where the side and the line connecting the endpoint of the
diagonal to the opposite base (The height of the trapezoid) as to form a right triangle is utilized

53
This document is not for sale.

In Rhombuses, we use the following formula:

The diagonals of a parallelogram are solved the same way as a trapezoid’s, utilizing the height of the figure as
well as one of the bases with the Pythagorean Theorem.

The calculation for the diagonals of a Kite uses the Pythagorean Theorem as well.

54
This document is not for sale.

Other Polygons also use the Pythagorean Theorem.

Angles

Exterior Angles: The angle formed outside the polygon. The sum of the exterior angles is always 360 degrees.

Interior Angles: Angles formed inside the polygon. The sum of the measurements of the angles of an n-gon is
(n-2)180.

55
This document is not for sale.

Calculations of interior and external angles through the properties angles: The measurement of an exterior
angle can be calculated based on its adjacent interior angle when one of the sides is extended seen as they are
then supplementary angles.

Exterior angles can also be calculated without the side extensions, what you have to do is subtract the
measurement of the interior angle to 360 degrees.

Parallel lines intersected by a transversal: When Parallel lines are intersected by a transversal, certain angles
with special relationships are formed.

 Alternate Interior Angles – Angles within the parallel lines on the opposite sides of the transversal are
always equal in measurement.

56
This document is not for sale.

 Alternate Exterior Angles – Angles outside the parallel lines on the opposite sides of the transversal are
always equal in measurement.

 Same Side Interior Angles – Angles on the same side of the transversal within the parallel lines are always
supplementary.

 Same Side Exterior Angles – Angles on the same side of the transversal outside the parallel lines are
always supplementary.

 Corresponding Angles – Angles on the same side of the transversal with one being in the exterior and one
in the interior are always equal.

57
This document is not for sale.

 Vertical Angles – Angles that are opposite each other in the intersection of straight lines are always equal in
measure.

 Linear Pair Angles – Two adjacent angles that form a straight line are always supplementary.

Triangles

Perimeter: To find the perimeter of a triangle, simply add the measurements of all the sides.

58
This document is not for sale.

Area: To find the area of a triangle, multiply the base to the height, then divide by 2.

Circles

Parts of a Circle: Circles have different parts.

 Center – The point in which all the other points in the circle are equidistant from.

 Chord – Line whose two endpoints are on the outer rim of a circle.

 Radius – The distance from the center to the circle’s outer rim.

 Diameter – A chord that passes through the center of a circle.

 Arc – A section of the circumference of a circle. Can be measured using the central angle or the
measurement of the arc itself. Denoted using the symbol ⌒

59
This document is not for sale.

 Sector – Portion of a circle enclosed by two radii and an arc. Denoted using the symbol ⌔

 Segment – Portion of a circle that is cut off by a chord or a secant. Denoted using the symbol ⌓

 Tangent – Line that intersects a circle in exactly one point.

 Secant – Line that intersects a circle in exactly two points.

 Point of Tangency – The intersection of a tangent and a circle.

Relationships of a Circle: Circles have different relationships.

 Tangent Circles – Two or more Circles that intersect at one point.

 Concentric Circles – Two or more Circles that share the same center with different radii.

 Congruent Circles – Two or more Circles with the same radius but different centers.

Circumference: The distance once around the circle. It can be calculated using the formula C=2r, where C is
the circumference, and r is the length of the radius. =3.1415926535897...

60
This document is not for sale.

Area: To calculate the area of the circle we use the formula A=r2.

Products of Chords: If two chords intersect in a circle, the product of the lengths of the segments of one chord
equal the product of the segments of the other.

Chord 1: (2)(12)=24; Chord 2: (3)(8) = 24

61
This document is not for sale.

Products of Secants: If two secant segments are drawn to a circle from the same external point, the product of
the length of one secant segment and its external part is equal to the product of the length of the other secant
segment and its external part.

Secant 1: (5+3)(3)=(8)(3)=24; Secant 2: (2+10)(2)=(12)(2)=24

Major and Minor Arcs: In the figure below, the portion of the circle from B to C is referred to as the minor arc
while the remainder of the circle is called the major arc.

62
This document is not for sale.

Minor Arcs measure less than a semicircle, Major arcs measure more than a semicircle.

Circular Sector: The area of a sector can be calculated using the formula A = ½ r2θ, where θ is the measurement of the
central angle in radians. If the central angle (θ) is in degrees, the following formula can be used A = πr2  (θ/360).

Circular Segment: Let R be the radius of the circle, c the chord length, s the arc length, h the height of the
segment, and d portion. The the height of the triangular area of the circular segment is equal to the area of the
circular sector minus the area of the triangular portion. In the illustration provided below, the circular segment
is represented by the yellow colored portion.

The radius is R = h + d = h/2+c2/8h

63
This document is not for sale.

The arc length is s = Rθ, where θ is in radians or s=arcsin(c/(h+c2/4h))(h+c2/4h)

The chord length is c=2Rsin(θ/2), where θ is in radians

The angle length is θ=2arctan(c/(2d))

The Sagitta is h=R(1-cos(θ /2)), where θ is in radians

The Area is equal to the area of the sector minus the are of the triangle or A=(r 2/2)(θ-sinθ), where θ is in
radians, or A=(R2/2)(()/180-sin θ), where θ is in degrees.

Secants and Tangents: We show circle O below in figure a. Points A, B, C, and D are on the circle. The
segments AP and DP are secants because they intersect the circle in two points. Notice that the arcs intercepted
are arcs CB and AD.

64
This document is not for sale.

How does the measure of angle P relate to the arcs CB and AD?

By drawing the segments DC and AB shown in red, we form the triangles ABP and DCP. These are similar
triangles because they have angle P in common and angles A and D must be equal because they are inscribed
angles intercepting the same arc, CB. This means that angles A and D must equal one half the measure of arc
CB.

In figure a, we also show angle 1 which is angle ACD because we will need to refer to it below. Notice that
angle 1 is inscribed and intercepts arc AD. Therefore, angle 1 has measure equal to one half of arc AD.

1. ANGLE outside a circle formed by two secants:

Below in figure b, we only show triangle DCP from the circle diagram shown in figure a above.

As shown in the diagram above, angle DCP is supplementary to angle 1.

65
This document is not for sale.

The three angles of triangle DCP must have a sum of 180°.

Solving this equation for angle P yeilds

This means that the measure of angle P, an angle external to a circle and formed by two secants, is equal to one
half the difference of the intercepted arcs.

2. ANGLE outside a circle formed by secants/tangents:

We just learned that the measure of an external angle P (as shown in figure b) when formed by two secants is
equal to one half the difference of the measures of the intercepted arcs.

In a related result, if one (or both) of the segments is tangent, as in segment PC in figure c shown below, the
external angle P is also one half the difference of the intercepted arcs DC and CB.

3. SEGMENTS formed by secants, drawn from a point, intersecting a circle:

Figure a is shown again for reference.

66
This document is not for sale.

We have already noted that triangles ABP and DCP are similar. This gives corresponding sides as follows:
PC ~ PB and PD ~ PA
In a proportion true for corresponding parts of similar triangles, we have

Notice that these are the products of the exterior part of each secant with each secant's entire length,

4. SEGMENTS formed by a secant and a tangent, drawn from a point, intersecting a circle:

In the case where one of the segments forming angle P is a tangent, we show figure c again.

67
This document is not for sale.

We have added segments CB and DC. Looking at triangles PCB and PDC, we have the following:

i. both triangles share angle P and


ii. angle D and angle PCB both have measure 1/2 arc CB, the intercepted arc

Thus, triangles PCB and PDC are similar. Since


sides PC ~ PD and
sides PB ~ PC
we can write the proportion

5. Far Arc – Near Arc Formula:

6. Secant-Tangent Formula:

If a secant segment and tangent segment are drawn to a circle from the same external point, the length of the
tangent segment is the geometric mean between the length of the secant segment and the length of the external
part of the secant segment.

Alternate Wording: ... the product of the length of the secant segment and its external part equals the square of
the length of the tangent segment.

68
This document is not for sale.

Degrees to Radians and Vice Versa: Angles can be measured using degrees or radians, so it is important we
know how to convert them if needed.

69
This document is not for sale.

General Equation and Graphing: The equation of a circle comes in two forms:

1) The standard form: (x-h)2 + (y-k)2 = r2

2) The general form : x2 + y2 + Dx + Ey + F = 0, where D, E, F are constants.

If the equation of a circle is in the standard form, we can easily identify the center of the circle, (h, k), and the
radius, r. Note: The radius, r, is always positive.

Example 1: (x-2)2 + (y-3)2 = 4. (a) Find the center and radius of the circle. (b) Graph the circle.

Note: A common mistake is to take h=-2 and K=-3. In an equation, if the sign preceding h and k, (h, k) are
negative, then h and k are positive. That is, h= 2 and k= 3.

Center: (h= 2, k= 3) = (2, 3) and radius r=2 since r2 = 4 => r = 4 = 2

If the equation is in the general form, we have to complete the square and bring the equation in the standard
form. Then, we can identify the center and radius correctly. We learned how to complete the square when
working with quadratic equations. We will review it through an example.

Example 2: x2 + y2 - 6x + 4y + 9 = 0. (a) Find the center and radius of the circle. (b) Graph the circle.

Completing the square:

 Write the equation in this form: (x2 - 6x + ?1) + (y2 + 4y + ?2) = -9 + ?1 + ?2. In the first parenthesis, we
group the x-terms and in the second the y-terms. The constant is moved on the right hand side. The question
mark, ?, is the number needed in each parenthesis to complete the square. Note that we have to add this
number to both sides of the equation. That is why you see ?1 and ?2, added to both sides.

 How to find the number to replace the question mark, ?1. Take the coefficient of x and divide it by 2, (-
6/2), and then square it, (-3)2 = 9. ?1 is going to be replaced by the number 9.

70
This document is not for sale.

 How to find the number to replace the question mark, ?2. Take the coefficient of y and divide it by 2, (4/2),
and then square it, (2)2 = 4. ?2 is going to be replaced by the number 4.

(x2 - 6x + ?1 ) + (y2 + 4y + ?2 ) = -9 + ?1 + ?2

(x2 - 6x + 9 ) + (y2 + 4y + 4 ) = -9 + 9 + 4

( x - 3 )2 + ( y + 2 )2 = 4

( x - 3 )2 + ( y - (-2) )2 = 4 This equation is in the standard form.

Center: (h= 3, k=-2) = ( 3, -2 ) and radius r=2 since r2 = 4 => r = 4 = 2

Additional Information

71
This document is not for sale.

72
This document is not for sale.

73
This document is not for sale.

74
This document is not for sale.

75
This document is not for sale.

76
This document is not for sale.

77
This document is not for sale.

78
This document is not for sale.

79
This document is not for sale.

80
This document is not for sale.

81
This document is not for sale.

82
This document is not for sale.

83
This document is not for sale.

Quadrilaterals

Perimeter: To get the Perimeter of a square or Rhombus, simply multiply the length of one side four times.
P=4a, where a is the length of one side. You could also use P=2d2 for squares, where d is the length of a
diagonal. For Rhombuses, you could also use P=2p2+q2, where p and q are the two diagonals.

To get the Perimeter of a Rectangle, use the formula P=2(l+w), where l is the length and w is the width. You
can also use P=2l+2d2-l2.

To get the Perimeter of a Parallelogram, use the formula P=2(a+b), where b is the base, and a is the side.

To get the perimeter of a trapezoid, simply add all the sides P=a+b+c+d, where a and b are the bases, and c and
d are the sides.

To get the perimeter of a kite, use the formula P=2(a+b), where a and b are the sides.

Quadrilateral Using Sides Using Diagonals

Square P=4a P=2d2

Rectangle P=2(l+w) P=2l+2d2-l2

Parallelogram P=2(a+b) Pythagorean

84
This document is not for sale.

Rhombus P=4a P=2p2+q2

Trapezoid P=a+b+c+d Pythagorean

Kite P=2(a+b) Pythagorean

Area: To find the area of a square, simply use the formula A=a 2, where a is the length of a side. You could also
use A=½ d2, where d is the length of the diagonals.

To find the area of a rectangle, use the formula A=wl, where l is the length and w is the width. You can also use
A=ld2-l2, where d is the diagonals and l is the length.

To find the area of a Parallelogram, use A=bh, where b is the length of the base, and h is the height.

To find the area of a Rhombus use A=pq/2, where p and q are the diagonals. Or, you could use A= ½ p4a2-p2,
where a is the side, and p is the diagonal.

To find the area of a Trapezoid, you can use the formula A=((a+b)/2)h, where a and b are the bases, and h is the
height.

To find the area of a Kite use A=pq/2, where p and q are the diagonals.

Quadrilateral Using Sides Using Diagonals

Square A=a2 A=½ d2

Rectangle A=wl A=ld2-l2

Parallelogram A=bh Pythagorean

Rhombus A= ½ p4a2-p2 A=pq/2

Trapezoid A=((a+b)/2)h Pythagorean

Kite Pythagorean A=pq/2

Parabola

Definition: A curve where any point is at an equal distance from a fixed point, the focus, and a fixed straight
line, the directrix. Parabolas, Hyperbolas, Circles, and Ellipses are what we call Conic Sections, or figures that
can be formed when a plane intersects with a cone. The angle of the plane with respect to the cone can change
the said figure.

85
This document is not for sale.

General Equation and Graphing: If a parabola has a vertical axis, the standard form of the equation of the
parabola is this: (x - h)2 = 4p(y - k), where p≠ 0. The vertex of this parabola is at (h, k). The focus is at (h, k +
p). The directrix is the line y = k - p. The axis is the line x = h. If p > 0, the parabola opens upward, and if p < 0,
the parabola opens downward. This graph is a function.

If a parabola has a horizontal axis, the standard form of the equation of the parabola is this: (y - k)2 = 4p(x - h),
where p≠ 0. The vertex of this parabola is at (h, k). The focus is at (h + p, k). The directrix is the line x = h - p.
The axis is the line y = k. If p > 0, the parabola opens to the right, and if p < 0, the parabola opens to the left.
Note that this graph is not a function.

86
This document is not for sale.

87
This document is not for sale.

The methods used here to rewrite the equation of a parabola into its standard form also apply when rewriting
equations of circles, ellipses, and hyperbolas. The standard forms for conic sections are factored forms that
allow you to immediately identify needed information. Different algebra situations call for different standard
forms — the form just depends on what you need from the equation.
For instance, if you want to convert the equation x2 + 10x – 2y + 23 = 0 into the standard form, you perform the
following steps, which contain a method called completing the square (a method you use to solve quadratic
equations):
1. Rewrite the equation with the x2 and x terms (or the y2 and y terms) on one side of the equation and the rest
of the terms on the other side.
x + 10x = 2y – 23
2

2. Add a number to each side to make the side with the squared term into a perfect square trinomial (thus
completing the square).
In this case, you add 25 to each side. x2 + 10x + 25 = 2y – 23 + 25 simplifies to x2 + 10x + 25 = 2y + 2.
3. Rewrite the perfect square trinomial in factored form, and factor the terms on the other side by the
coefficient of the variable.
(x + 5)2 = 2(y + 1)
Find the standard equation of the parabola with vertex at (4, 2) and focus at (4, -3).

Step 1: Determine the following:

➢ the coordinates of the vertex (h, k).

➢ the orientation of the axis.

➢ the distance between the focus and the vertex (p).

Vertex:

(h, k) = (4, 2) Given

88
This document is not for sale.

Axis orientation: Both the focus and the vertex fall on the axis. Looking at their coordinates reveals that both
fall on the vertical line x = 4. Thus the axis of the parabola is vertical.

Distance between focus and vertex: Since both focus and vertex fall on the vertical line x = 4 determine the
distance between the point by subtracting their y-coordinates and taking the absolute value.

| y f − y v |=| −3−2 |=| −5 |=5=p

Step 2: Substitute the values for h, k and p into the equation for a parabola with a vertical axis.

(x - h)2 = 4p(y - k) Vertical axis

( x−4 ) 2 =4( 5 )( y−2 ) Sub

( x−4 ) 2 =20( y−2 ) Simplify

Ellipse

Definition: In mathematics, an ellipse is a plane curve surrounding two focal points, such that for all points on
the curve, the sum of the two distances to the focal points is a constant. As such, it generalizes a circle, which is
the special type of ellipse in which the two focal points are the same.

89
This document is not for sale.

General Equation and Graphing: For all (x, y), d1 + d2 = constant. When talking about an ellipse, the
following terms are used:

 The foci are two fixed points equidistant from the center of the ellipse.

 The vertices are the points on the ellipse that fall on the line containing the foci.

 The line segment or chord joining the vertices is the major axis.

 The midpoint of the major axis is the center.

 The axis perpendicular to the major axis is the minor axis.

STANDARD EQUATION OF AN ELLIPSE:

➢ Center coordinates (h, k)

➢ Major axis 2a

➢ Major axis 2b

➢0<b<a

( x−h ) 2 / a 2 + ( y−k ) 2 / b 2 = 1 major axis is horizontal

( x−h ) 2 / b 2 + ( y−k ) 2 / a 2 = 1 major axis is vertical

The foci lie on the major axis, c units from the center, with c2 = a2 - b2.

Find the standard equation of the ellipse having foci at (2, 1) and (-4, 1) and a minor axis of 10.

Step 1: Determine the following:

➢ The coordinates of the center (h, k).

90
This document is not for sale.

➢ The distance of half the minor axis (b).

➢ The length of half the major axis (a).

➢ The orientation of the major axis.

Center: Since the foci are equidistant from the center of the ellipse the center can be determine by finding the
midpoint of the foci.

( h, k )=( (2+( −4 )) 2 , (1+1) 2 )=( − 2 2 , 2 2 )=( −1,1 )

Length of b: The minor axis is given as 10, which is equal to 2b.

2b=10→b=5

Length of a: To find a the equation c2 = a2 + b2 can be used but the value of c must be determined. Since c is the
distance from the foci to the center, take either foci and determine the distance to the center. Then solve for a.

Foci (2, 1): c=| 2−( −1 ) |=| 3 |=3

Foci (-4, 1): c=| −4− ( −1 ) |=| −3 |=3

c=3

c 2 = a 2 − b 2 →a= c 2 + b 2

a= 3 2 + 5 2 →a= 34

Orientation of major axis: Since the two foci fall on the horizontal line y = 1, the major axis is horizontal.

Step 2: Substitute the values for h, k, a and b into the equation for an ellipse with a horizontal major axis.

Horizontal major axis equation:

( x−h ) 2 / a 2 + ( y−k ) 2 / b 2 =1

Substitute values:

[ x−( −1 ) ] 2 / 34 2 + ( y−1 ) 2 / 5 2 =1

Simplify:

( x+1 ) 2 / 34 2 + ( y−1 ) 2 / 5 2 =1

91
This document is not for sale.

Hyperbola

Definition: A curve with two branches that are symmetrical in any case. Any point P is closer to F1 than to F2
by some constant amount.

92
This document is not for sale.

General Equation and Graphing: By placing a hyperbola on an x-y graph (centered over the x-axis and y-
axis), the equation of the curve is:

x2a2 − y2b2 = 1

Also:

One vertex is at (a, 0), and the other is at (−a, 0)

The asymptotes are the straight lines:

 y = (b/a)x
 y = −(b/a)x

(Note: the equation is similar to the equation of the ellipse: x2/a2 + y2/b2 = 1, except for a "−" instead of a "+")

93
This document is not for sale.

The graph of a hyperbola is not continuous--every hyperbola has two distinct branches. The line segment
containing both foci of a hyperbola whose endpoints are both on the hyperbola is called the transverse axis. The
endpoints of the transverse axis are called the vertices of the hyperbola. The point halfway between the foci (the
midpoint of the transverse axis) is the center.

The standard equation for a hyperbola with a horizontal transverse axis is - = 1. The center is
at (h, k). The distance between the vertices is 2a. The distance between the foci is 2c. c2 = a2 + b2. The line
segment of length 2b perpendicular to the transverse axis whose midpoint is the center is the conjugate axis of
the hyperbola.
The standard equation for a hyperbola with a vertical transverse axis is - = 1. The center is
at (h, k). The distance between the vertices is 2a. The distance between the foci is 2c. c = a2 + b2.
2

Every hyperbola has two asymptotes. A hyperbola with a horizontal transverse axis and center at (h, k) has one
asymptote with equation y = k + (x - h) and the other with equation y = k - (x - h). A hyperbola with a
vertical transverse axis and center at (h, k) has one asymptote with equation y = k + (x - h) and the other with
equation y = k - (x - h).
The eccentricity of a hyperbola, like an ellipse, is e = . For all hyperbolas, though, c > a, so e > 1. If e is close
to one, the branches of the hyperbola are very narrow, but if e is much greater than one, then the branches of the
hyperbola are very flat.

94
This document is not for sale.

95
This document is not for sale.

Solid Geometry

Solid Geometry is the study of 3d shapes.

96
This document is not for sale.

97
This document is not for sale.

Definition and Types: There are different Types of Solids.

 Cube: Solid bounded by six square faces.

 Rectangular Prism: Solid bounded by six quadrilateral faces, typically consisting of rectangles.

 Triangular Prism: Solid bounded by three quadrilateral faces and two triangular faces.

 Square Based Pyramid: Solid bounded by four triangular faces and a square base.

 Triangle Based Pyramid: Solid Bounded by four Triangular faces.

 Sphere: A circle, but in a three dimensional space.

 Cylinder: Also known as a circular prism.

 Cone: A three-dimensional geometric shape that tapers smoothly from a flat base to a point called the apex
or vertex.

 Oblique Prisms: Prisms that have congruent bases but don’t necessarily have them align exactly with each
other.

 Other Prisms: Includes Pentagonal Prisms, Hexagonal Prisms, Octagonal Prisms, etc.

98
This document is not for sale.

Surface Area: Surface area is the total area that the surface of a solid occupies.

 Cube: A=6a2, where a is the length of one edge.

 Rectangular Prism: A=2(wl+hl+hw), where l is the length, w is the width, and h is the height.

 Triangular Prism: A=bh+2ls+lb, where b is the base, h is the height, l is the length, and s is the side. Or
A=bh+L(s1+s2+s3), where s are the sides of the triangle face, b is the base, L is the length, and h is the
height. Or A=2b+ph, where b is the area of a base, p is the perimeter of a base, and h is the height.

 Square Based Pyramid: A=a2+2a((a2/4)+h2), where a is the length of the base edge, and h is the height.

99
This document is not for sale.

 Triangle Based Pyramid: A=A+(3/2)bh, where A is the area of the pyramid’s base, b is the vase of one of
the faces, and h is the slant height of one of the faces.

 Sphere: A=4r2, where r is the radius.

 Cylinder: A=2rh+2r2, where r is the radius and h is the height.

 Cone: A=r(r+h2+r2), where r is the radius, and h is the height.

 Oblique Prisms: Find the areas of all the faces then add them.

 Hexagonal Prism: A=6ah+33a2, where a is the base edge, and h is the height.

 Pentagonal Prism: A=5ah+ ½ a2 5(5+25), where a is the base edge, and h is the height.

Volume: The 3 dimensional space enclosed by the solid.

 Cube: V=a3, where a is the length of one edge.

 Rectangular Prism: V=whl, where l is the length, w is the width, and h is the height.

 Triangular Prism: V= ¼ h﹣a4+2(ab)2+2(ac)2﹣b4+2(bc)2﹣c4, where a b and c are the base sides, and h is
the height.

 Square Based Pyramid: V=a2(h/3), where a is the base edge, and h is the height.

100
This document is not for sale.

 Triangle Based Pyramid: V= 1/3 AH, where A is the area of the triangle base, and H is the height.

 Sphere: V=(4/3)r3, where r is the radius.

 Cylinder: V=r2h, where r is the radius, and h is the height.

 Cone: V=r2(h/3), where r is the radius, and h is the height.

 Oblique Prisms: V=bh, where b is the base area, and h is the height.

 Hexagonal Prism: V=((33)/2)a2h, where a is the base edge, and h is the height.

 Pentagonal Prism: V=1/4(5(5+25))a2h, where a is the base edge, and h is the height.

Factoring
Factoring is the process of finding the factors of a certain algebraic expression.

Methods of Factoring

Common Factor: Find the common factor of an expression then apply the distributive property, but in reverse.

Ex. 3y2+12y

The common factor is 3y, since both terms can be exactly divided by 3y.

Therefore, the factors of this expression are (3y)(y+4)

Special Binomial Products: Apply your knowledge in Special Binomial Products, then reverse the process.

Ex. 4x2-9

The expression is a difference of squares special binomial product, which follows the formula (a+b)(a-b)=a2-b2.
Therefore, the factors of this expression are (2x+3)(2x-3).

Other Special Products to take note include

a2 − b2 = (a+b)(a−b)

a2 + 2ab + b2 = (a+b)(a+b)

a2 − 2ab + b2 = (a−b)(a−b)

a3 + b3 = (a+b)(a2−ab+b2)

a3 − b3 = (a−b)(a2+ab+b2)

101
This document is not for sale.

a3+3a2b+3ab2+b3 = (a+b)3

a3−3a2b+3ab2−b3 = (a−b)3
Mixed: Apply the earlier methods.

Ex. w4-16

An exponent of 4? Maybe we could try an exponent of 2:

w4 − 16 = (w2)2 − 42

Yes, it is the difference of squares

w4 − 16 = (w2 + 4)(w2 − 4)

And "(w2 − 4)" is another difference of squares

w4 − 16 = (w2 + 4)(w + 2)(w − 2)

Ex. 3u4-24uv3

Remove common factor "3u":

3u4 − 24uv3 = 3u(u3 − 8v3)

Then a difference of cubes:

3u4 − 24uv3 = 3u(u3 − (2v)3)

= 3u(u−2v)(u2+2uv+4v2)

Ex. z3-z2-9z+9

Try factoring the first two and second two separately:

z2(z−1) − 9(z−1)

Wow, (z-1) is on both, so let us use that:

(z2−9)(z−1)

And z2−9 is a difference of squares

(z−3)(z+3)(z−1)

102
This document is not for sale.

Hypotheses and Arguments


Hypotheses and Arguments are important for proving the various properties, theorems, and postulates in
Geometry. The following is an example of proof for same side interior angles.

Definitions

Definitions are also self explanatory statements. The following includes some common additional definitions.

 Congruent Triangles - Triangles in which corresponding parts (sides and angles) are equal in measure

 Similar Triangles - Triangles in which corresponding angles are equal in measure and corresponding sides
are in proportion (ratios equal)

 Angle Bisector - A ray that begins at the vertex of an angle and divides the angle into two angles of equal
measure

103
This document is not for sale.

 Segment Bisector - A ray, line or segment that divides a segment into two parts of equal measure

 Legs of an Isosceles Triangle - The sides of equal measure in an isosceles triangle

 Base of an Isosceles Triangle - The third side of an isosceles triangle

 Equiangular - Having angles that are all equal in measure

 Perpendicular Bisector - A line that bisects a segment and is perpendicular to it

 Altitude - A segment from a vertex of a triangle perpendicular to the line containing the opposite side

 Sine - For an acute angle of a right triangle, the ratio of the side opposite the angle to the measure of the
hypotenuse. (opp/hyp)

 Cosine - For an acute angle of a right triangle the ratio of the side adjacent to the angle to the measure of the
hypotenuse. (adj/hyp)

 Tangent - For an acute angle of a right triangle, the ratio of the side opposite to the angle to the measure of
the side adjacent (opp/adj)

 Incircle – An inscribed circle of a polygon. The largest circle completely within a polygon.

 Excircle – Largest circles formed by the extensions of the sides of a polygon.

 Circumcircle – Circle that passes through each of the triangle’s vertices.

104
This document is not for sale.

 Incenter – The center of the incircle.

 Circumcenter – Center of the Circumcircle

 Centroid – The geometric center of a figure. The mean position of all points in a figure. The point of
intersection of the three medians.

 Inscribed Angle – Angle formed in the interior of a circle when a secants and tangents intersect.

105
This document is not for sale.

 Right Angle – All right angles are congruent.

 Straight Angle – All straight angles are congruent.

 Line – A line contains at least two points. Through any two points, there is exactly one line. If two line
intersect, they intersect in exactly one point. If two line intersect, then exactly one plane contains both lines.

 Plane – Contains at least three non collinear points. Through any three noncollinear points, there is exactly
one plane. If two points lie in a plane, the line joining them lies in that plane. If two planes intersect, then
their intersection is a line. If appoint lies outside a line, then exactly one plain contains both the line and the
point.

 Parallelogram – If one pair of sides of a quadrilateral is both parallel and congruent, the quadrilateral is a
parallelogram.

 Parallelograms Sides – If a quadrilateral is a parallelogram, the opposite sides are parallel. If a


quadrilateral is a parallelogram, the opposite sides are congruent. And Vice Versa

 Parallelograms Angles – If a quadrilateral is a parallelogram, the opposite angles are congruent. If a


quadrilateral is a parallelogram, the consecutive angles are supplementary. And Vice Versa.

 Parallelograms Diagonals – If a quadrilateral is a parallelogram, the diagonals bisect each other. If a


quadrilateral is a parallelogram, the diagonals form two congruent triangles. And Vice Versa.

 Rectangle – If a parallelogram has one right angle, it is a rectangle. A parallelogram is a rectangle if and
only if its diagonals are congruent. A rectangle is a parallelogram with four right angles.

 Rhombus – A rhombus is a parallelogram with four congruent sides. If a parallelogram has two consecutive
sides congruent, it is a rhombus. A parallelogram is a rhombus if and only if each diagonal bisects a pair of
opposite angles. A parallelogram is a rhombus if and only if the diagonals are perpendicular.

 Square – A square is a parallelogram with four congruent sides and four right angles. A quadrilateral is a
square if and only if it is a rhombus and a rectangle.

 Trapezoid – A trapezoid is a quadrilateral with exactly one pair of parallel sides.

 Isosceles Trapezoid – An isosceles trapezoid is a trapezoid with congruent legs. A trapezoid is isosceles if
and only if the base angles are congruent. A trapezoid is isosceles if and only if the diagonals are congruent.
If a trapezoid is isosceles, the opposite angles are supplementary.

 Radius – In a circle, a radius perpendicular to a chord bisects the chord and the arc. In a circle, a radius that
bisects a chord is perpendicular to the chord. In a circle, the perpendicular bisector of a chord passes through
the center of the circle. If a line is tangent to a circle, it is perpendicular to the radius drawn to the point of
tangency.

 Chords – In a circle, or congruent circles, congruent chords are equidistant from the center (and converse).
In a circle, or congruent circles, congruent chords have congruent arcs (and converse). In a circle, parallel

106
This document is not for sale.

chords intercept congruent arcs. In the same circle, or congruent circles, congruent central angles have
congruent chords (and converse).

 Tangents – Tangent segments to a circle from the same external point are congruent.

 Arcs – In the same circle, or congruent circles, congruent central angles have congruent arcs (and converse).

 Circle Angles – An angle inscribed in a semicircle is a right angle. In a circle, inscribed angles that
intercept the same arc are congruent. The opposite angles in a cyclic quadrilateral are supplementary. In a
circle, or congruent circles, congruent central angles have congruent arcs.

Theorems

Theorems are mathematical statement that is non-self-evident. Despite this, theorems can be proven to be true
and are therefore, generally accepted statements.

Theorems on Lines

 Common Segments Theorem - Given collinear points A,B,C and D arranged as shown, if line AB  line
CD then line AC  line BC

 Alternate Interior Angles Theorem - If two parallel lines are intersected by a transversal, then alternate
interior angles are equal in measure

 Alternate Exterior Angles Theorem - If two parallel lines are intersected by a transversal, then alternate
exterior angles are equal in measure

 Same Side Interior Angles Theorem - If two parallel lines are intersected by a transversal, then same-side
interior angles are supplementary.

 Converse of Alternate Interior Angles Theorem - If two lines are intersected by a transversal and
alternate interior angles are equal in measure, then the lines are parallel

 Converse of Alternate Exterior Angles Theorem - If two lines are intersected by a transversal and
alternate exterior angles are equal in measure, then the lines are parallel

 Converse of Same Side Interior Angles Theorem - If two lines are intersected by a transversal and same-
side interior angles are supplementary, then the lines are parallel

 Theorem 1 - If two intersecting lines form a linear pair of congruent angles, then the lines are perpendicular

 Theorem 2 - If two lines are perpendicular to the same transversal, then they are parallel

 Perpendicular Transversal Theorem - If a transversal is perpendicular to one of two parallel lines, then it
is perpendicular to the other one

 Perpendicular Bisector Theorem - If a point is on the perpendicular bisector of a segment, then it is


equidistant from the endpoints of the segment

107
This document is not for sale.

 Converse of the Perpendicular Bisector Theorem - If a point is the same distance from both the endpoints
of a segment, then it lies on the perpendicular bisector of the segment

 Parallel Lines Theorem - In a coordinate plane, two nonvertical lines are parallel IF they have the same
slope.

 Perpendicular Lines Theorem - In a coordinate plane, two nonvertical lines are perpendicular IFF the
product of their slopes is -1.

 Two-Transversals Proportionality Corollary - If three or more parallel lines intersect two transversals,
then they divide the transversals proportionally.

Theorems on Angles

 Linear Pair Theorem - If two angles form a linear pair, then they are supplementary.

 Congruent Supplements Theorem - If two angles are supplements of the same angle, then they are
congruent.

 Congruent Complements Theorem - If two angles are complements of the same angle, then they are
congruent.

 Right Angle Congruence Theorem - All right angles are congruent.

 Vertical Angles Theorem - Vertical angles are equal in measure

 Theorem 1 - If two congruent angles are supplementary, then each is a right angle.

 Angle Bisector Theorem - If a point is on the bisector of an angle, then it is equidistant from the sides of
the angle.

 Converse of the Angle Bisector Theorem - If a point in the interior of an angle is equidistant from the
sides of the angle, then it is on the bisector of the angle.

Theorems on Triangles

 CPCTC – Corresponding Parts of congruent triangles are congruent.

 Sum of Two Sides- The sum of the lengths of any two sides of a triangle must be greater than the third side.

 Longest side – In a triangle, the longest side is across from the largest angle and vice versa.

 Side-side-side (SSS) Similarity Theorem - If the three sides of one triangle are proportional to the three
corresponding sides of another triangle, then the triangles are similar.

 Side-angle-side (SAS) Similarity Theorem - If two sides of one triangle are proportional to two sides of
another triangle and their included angles are congruent, then the triangles are similar.

108
This document is not for sale.

 Third Angles Theorem - If two angles of one triangle are congruent to two angles of another triangle, then
the third pair of angles are congruent

 Triangle Sum Theorem - The sum of the measure of the angles of a triangle is 180

 Corollary 1 - The acute angles of a right triangle are complementary.

 Exterior Angle Theorem - An exterior angle of a triangle is equal in measure to the sum of the measures of
its two remote interior angles.

 Triangle Proportionality Theorem - If a line parallel to a side of a triangle intersects the other two sides,
then it divides those sides proportionally.

 Converse of Triangle Proportionality Theorem - If a line divides two sides of a triangle proportionally,
then it is parallel to the third side.

 Triangle Angle Bisector Theorem - An angle bisector of a triangle divides the opposite sides into two
segments whose lengths are proportional to the lengths of the other two sides.

 Angle-Angle-Side (AAS) Congruence Theorem - If two angles and a non-included side of one triangle are
equal in measure to the corresponding angles and side of another triangle, then the triangles are congruent.

 Hypotenuse-Leg (HL) Congruence Theorem - If the hypotenuse and a leg of a right triangle are congruent
to the hypotenuse and a leg of another right triangle, then the triangles are congruent.

 Isosceles Triangle Theorem - If two sides of a triangle are equal in measure, then the angles opposite those
sides are equal in measure

 Converse of Isosceles Triangle Theorem - If two angles of a triangle are equal in measure, then the sides
opposite those angles are equal in measure

 Corollary 2 - If a triangle is equilateral, then it is equiangular

 Corollary 3 - The measure of each angle of an equiangular triangle is 60

 Corollary 4 - If a triangle is equiangular, then it is also equilateral

 Theorem 1 - If the altitude is drawn to the hypotenuse of a right triangle, then the two triangles formed are
similar to the original triangle and to each other.

 Pythagorean Theorem - In any right triangle, the square of the length of the hypotenuse is equal to the sum
of the square of the lengths of the legs.

 Geometric Means Corollary A - The length of the altitude to the hypotenuse of a right triangle is the
geometric mean of the lengths of the two segments of the hypotenuse.

 Geometric Means Corollary B - The length of a leg of a right triangle is the geometric mean of the lengths
of the hypotenuse and the segment of the hypotenuse adjacent to that leg.

109
This document is not for sale.

 Circumcenter Theorem - The circumcenter of a triangle is equidistant from the vertices of the triangle.

 Incenter Theorem - The incenter of a triangle is equidistant from the sides of the triangle.

 Centroid Theorem - The centriod of a triangle is located 2/3 of the distance from each vertex to the
midpoint of the opposite side.

 Triangle Midsegment Theorem - A midsegment of a triangle is parallel to a side of triangle, and its length
is half the length of that side.

 Theorem 2 - If two sides of a triangle are not congruent, then the larger angle is opposite the longer side.

 Theorem 3 - If two angles of a triangle are not congruent, then the longer side is opposite the larger angle.

 Triangle Inequality Theorem - The sum of any two side lengths of a triangle is greater than the third side
length.

 Hinge Theorem - If two sides of one triangle are congruent to two sides of another triangle and the third
sides are not congruent, then the longer third side is across from the larger included angle.

 Converse of Hinge Theorem - If two sides of one triangle are congruent to two sides of another triangle
and the third sides are not congruent, then the larger included angle is across from the longer third side.

 Converse of Pythagorean Theorem - If the sum of the squares of the lengths of two sides of a triangle is
equal to the square of the length of the third side, then the triangle is a right triangle.

 Pythagorean Inequalities Theorem - In ¨ABC, c is the length of the longest side. If c² > a² + b², then ¨ABC
is an obtuse triangle. If c² < a² + b², then ¨ABC is acute.

 45 45 90 Triangle Theorem - In a 45˚-45˚-90˚ triangle, both legs are congruent, and the length of the
hypotenuse is the length of a length times the square root of 2.

 30 60 90 Theorem - In a 30˚-60˚-90˚ triangle, the length of the hypotenuse is 2 times the length of the
shorter leg, and the length of the longer leg is the length of the shorter leg times the square root of 3.

Theorems on other Polygons

 Polygon Angle Sum Theorem - The sum of the interior angle measures of a convex polygon with n sides.

 Polygon Exterior Angle Sum Theorem - The sum of the exterior angle measures, one angle at each vertex,
of a convex polygon is 360˚.

 Theorem 1 - If a quadrilateral is a parallelogram, then its opposite sides are congruent.

 Theorem 2 - If a quadrilateral is a parallelogram, then its opposite angles are congruent.

 Theorem 3 - If a quadrilateral is a parallelogram, then its consecutive angles are supplementary.

110
This document is not for sale.

 Theorem 4 - If a quadrilateral is a parallelogram, then its diagonals bisect each other.

 Theorem 5 - If one pair of opposite sides of a quadrilateral are parallel and congruent, then the quadrilateral
is a parallelogram.

 Theorem 6 - If both pairs of opposite sides of a quadrilateral are congruent, then the quadrilateral is a
parallelogram.

 Theorem 7 - If both pairs of opposite angles are congruent, then the quadrilateral is a parallelogram.

 Theorem 8 - If an angle of a quadrilateral is supplementary to both of its consecutive angles, then the
quadrilateral is a parallelogram.

 Theorem 9 - If the diagonals of a quadrilateral bisect each other, then the quadrilateral is a parallelogram.

 Theorem 10 - If a quadrilateral is a rectangle, then it is a parallelogram.

 Theorem 11 - If a parallelogram is a rectangle, then its diagonals are congruent.

 Theorem 12 - If a quadrilateral is a rhombus, then it is a parallelogram.

 Theorem 13 - If a parallelogram is a rhombus then its diagonals are perpendicular.

 Theorem 14 - If a parallelogram is a rhombus, then each diagonal bisects a pair of opposite angles.

 Theorem 15 - If one angle of a parallelogram is a right angle, then the parallelogram is a rectangle.

 Theorem 16 - If the diagonals of a parallelogram are congruent, then the parallelogram is a rectangle.

 Theorem 17 - If one pair of consecutive sides of a parallelogram are congruent, then the parallelogram is a
rhombus.

 Theorem 18 - If the diagonals of a parallelogram are perpendicular, then the parallelogram is a rhombus.

 Theorem 19 - If one diagonal of a parallelogram bisects a pair of opposite angles, then the parallelogram is
a rhombus.

 Theorem 20 - If a quadrilateral is a kite then its diagonals are perpendicular.

 Theorem 21 - If a quadrilateral is a kite then exactly one pair of opposite angles are congruent.

 Theorem 22 - If a quadrilateral is an isosceles trapezoid, then each pair of base angles are congruent.

 Theorem 23 - If a trapezoid has one pair of congruent base angles, then the trapezoid is isosceles.

 Theorem 24 - A trapezoid is isosceles if and only if its diagonals are congruent.

111
This document is not for sale.

 Trapezoid Midsegment Theorem - The midsegment of a trapezoid is parallel to each base, and its length is
one half the sum of the lengths of the bases.

 Proportional Perimeters and Areas Theorem – If the similarity ratio of two figure is a/b, then the ratio of
their perimeter is a/b and the ratio of their areas is a 2/b2 or (a/b)2

Theorems on Circles

 Theorem 1 - If a line is tangent to a circle, then it is perpendicular to the radius drawn to the point of
tangency.

 Theorem 2 - If a line is perpendicular to a radius of a circle at a point on the circle, then the line is tangent
to the circle.

 Theorem 3 - If two segments are tangent to a circle from the same external point then the segments are
congruent.

 Theorem 4 - In a circle or congruent circles: congruent central angles have congruent chords, congruent
chords have congruent arcs and congruent arcs have congruent central angles.

 Theorem 5 - In a circle, if a radius (or diameter) is perpendicular to a chord, then it bisects the chord and its
arc.

 Theorem 6 - In a circle, the perpendicular bisector of a chord is a radius (or diameter).

 Inscribed Angle Theorem - The measure of an inscribed angle is half the measure of its intercepted arc.

 Corollary 1 - If inscribed angles of a circle intercept the same arc or are subtended by the same chord or
arc, then the angles are congruent

 Theorem 7 - An inscribed angle subtends a semicircle if the angle is a right angle

 Theorem 8 - If a quadrilateral is inscribed in a circle, then its opposite angles are supplementary.

 Theorem 9 - If a tangent and a secant (or chord) intersect on a circle at the point of tangency, then the
measure of the angle formed is half the measure of its intercepted arc.

 Theorem 10 - If two secants or chords intersect in the interior of a circle, then the measure of each angle
formed is half the sum of the measures of the intercepted arcs.

 Theorem 11 - If a tangent and a secant, two tangents or two secants intersect in the exterior of a circle, then
the measure of the angle formed is half the difference of the measure of its intercepted arc.

 Chord-Chord Product Theorem - If two chords intersect in the interior of a circle, then the products of the
lengths of the segments of the chords are equal.

112
This document is not for sale.

 Secant-Secant Product Theorem - If two secants intersect in the exterior of a circle, then the product of
the lengths of one secant segment and its external segment equals the product of the lengths of the other
secant segment and its external segment.

 Secant-Tangent Product Theorem - If a secant and a tangent intersect in the exterior of a circle, then the
product of the lengths of the secant segment and its external segment equals the length of the tangent
segment squared.

 Equation of a Circle - The equal of a circle with center (h, k) and radius r is (x-h)2+(y-k)2=r2

Properties and Postulates

Properties are anything that applies to any given set. They are typically always a given when it comes to
proving, since they are the foundation of mathematics. These statements are self evident. (See Properties of
Numbers). Postulates are statements that don’t require proof since they are already taken as true.

Postulates on Algebra

 Addition Property of Equality - If the same number is added to equal numbers, then the sums are equal

 Subtraction Property of Equality - If the same number is subtracted from equal numbers, then the
differences are equal

 Multiplication Property of Equality - If equal numbers are multiplied by the same number, then the
products are equal

 Division Property of Equality - If equal numbers are divided by the same number, then the quotients are
equal

 Reflexive Property of Equality - A number is equal to itself

 Symmetric Property of Equality - If a = b then b = a

 Substitution Property of Equality - If values are equal, then one value may be substituted for the other.

 Transitive Property of Equality - If a = b and b = c then a = c

 Distributive Property - a(b + c) = ab + ac

 Reflexive Property of Congruence - A  A

 Symmetric Property of Congruence - If A  B, then B  A

 Transitive Property of Congruence - If A  B and B  C then A  C

Postulates on Lines

113
This document is not for sale.

 Segment Addition Postulate - For any segment, the measure of the whole is equal to the sum of the
measures of its non-overlapping parts

 Postulate 1 - Through any two points there is exactly one line

 Postulate 2 - If two lines intersect, then they intersect at exactly one point.

 Corresponding Angles Postulate - If two parallel lines are intersected by a transversal, then the
corresponding angles are equal in measure

 Converse of Corresponding Angles Postulate - If two lines are intersected by a transversal and
corresponding angles are equal in measure, then the lines are parallel

 Postulate 3 - Through a point not on a given line, there is one and only one line parallel to the given line

Postulates on Angles

 Angle Addition Postulate - For any angle, the measure of the whole is equal to the sum of the measures of
its non overlapping parts

Postulates on Triangles

 Angle-Angle (AA) Similarity Postulate - If two angles of one triangle are equal in measure to two angles
of another triangle, then the two triangles are similar

 Side-Angle-Side (SAS) Congruence Postulate - If two sides and the included angle of one triangle are
equal in measure to the corresponding sides and angle of another triangle, then the triangles are congruent.

 Side-Side-Side (SSS) Congruence Postulate - If three sides of one triangle are equal in measure to the
corresponding sides of another triangle, then the triangles are congruent

 Angle-Side-Angle (ASA) Congruence Postulate - If two angles and the included side of one triangle are
congruent to two angles and the included side of another triangle, then the triangles are congruent.

 Law of Sines - For any triangle ABC with side lengths a, b, and c, sinA/a=sinB/b=sinC/c

 Law of Cosines – For any triangle, ABC with sides a, b, and c, a 2=b2+c2-2bc cosA, b2=a2+c2-2ac cosB,
c2=a2+b2-2ac cosC

 Altitude Rule – The altitude to the hypotenuse of a right triangle is the mean proportional between the
segments into which it divides the hypotenuse.

 Leg Rule – Each leg of a right triangle is the mean proportional between the hypotenuse and the projection
of the leg on the hypotenuse.

Postulates on Plane

 Postulate 1 - Through any three noncollinear points there is exactly one plane containing them.

114
This document is not for sale.

 Postulate 2 - If two points lie in a plane, then the line containing those points lies in the plane

Postulates on Polygons

 Area Addition Postulate - The area of a region is equal to the sum of the areas of its nonoverlapping parts.

Postulates on Circles

 Arc Addition Postulate - The measure of an arc formed by two adjacent arcs is the sum of the measures of
the two arcs.

Linear Equations and Inequalities


Linear Equation

Linear Equations are equations with a degree of 1, and gives a straight line when plotted on a graph.

Linear Equations can take the form of y=mx+b or the slope intercept form, where m is the slope, and b is the y
intercept of the line. The slope always follows m=rise (vertical change)/run (horizontal change), so if a line rises
by one unit every two unites (run), then the value of m is ½.

They can also take the point slope form, or y-y1=m(x-x1), where m is the slope and x1 and y1 are points on the
line.

115
This document is not for sale.

Linear Equations can also take the general form of Ax+By+C=0, where A and B cannot be 0.

How to find the equation of the line from two points.

1. Find the slope of the line using the coordinates of the point. Find the rise and run.

2. Put the slope and one point into the "Point-Slope Formula" formula

3. Simplify to get the Point-Intercept formula

How to find the midpoint of a Linear Graph

1. Add both "x" coordinates, divide by 2

2. Add both "y" coordinates, divide by 2

116
This document is not for sale.

Or M=((XA+XB)/2, (YA+YB)/2)

How to find the Intercepts

1. X-intercept: point where the graph crosses the x-axis (where y = 0). To find x-intercept, simply let y = 0, then
find x.

2. Y-intercept: point where the graph crosses the y-axis (where x = 0). To find y-intercept, simply let x = 0, then
find y.

We can find out if two lines are parallel if they have the same slope.

We can find out if two lines are perpendicular when one line has a slope of m and the perpendicular line has a
slope of -1/m, or in short, when the perpendicular line has a negative reciprocal slope.

Linear Inequality

Linear Inequalities are like linear equations, but instead of an equal sign, they use inequality signs and form the
graph of a line with a shaded side.

First, lets learn how to solve general inequalities.

Our aim is to have x (or whatever the variable is) on its own on the left of the inequality sign:

Something like x < 5 or y ≥ 11

We call that "solved".

Example: x + 2 > 12

Subtract 2 from both sides:

117
This document is not for sale.

x + 2 − 2 > 12 − 2

Simplify:

x > 10

Now let us take a look at linear inequalities.

First, graph the "equals" line, then shade in the correct area.

There are three steps:

 Rearrange the equation so "y" is on the left and everything else on the right.
 Plot the "y=" line (make it a solid line for y≤ or y≥, and a dashed line for y< or y>)
 Shade above the line for a "greater than" (y> or y≥)
or below the line for a "less than" (y< or y≤).

Let us try some examples:

Example: y≤2x-1

1. The inequality already has "y" on the left and everything else on the right, so no need to rearrange

2. Plot y=2x-1 (as a solid line because y≤ includes equal to)

3. Shade the area below (because y is less than or equal to)

118
This document is not for sale.

Example: y/2 + 2 > x

1. We will need to rearrange this one so "y" is on its own on the left:

 Start with: y/2 + 2 > x


 Subtract 2 from both sides: y/2 > x − 2
 Multiply all by 2: y > 2x − 4

2. Now plot y = 2x − 4 (as a dashed line because y> does not include equals to)

3. Shade the area above (because y is greater than)

The dashed line shows that the inequality does not include the line y=2x-4.

119
This document is not for sale.

Linear Function

Linear Equations can be written as functions. f(x) = mx+b. There is also what is called the Identity function,
which is f(x)=x. When graphed, this function has a line that is tilted 45 degrees.

Quadratic Equations and Inequalities


Quadratic Equation and Function

Quadratic Equations are equations that have a degree of 2, and form a parabola or a curve when graphed.

Quadratic Equations take the form ax2+bx+c=0.

The Quadratic Formula is used to find solutions to Quadratic Equations.

120
This document is not for sale.

The Quadratic Formula has two answers.

But it does not always work out like that!

 Imagine if the curve "just touches" the x-axis.

 Or imagine the curve is so high it doesn't even cross the x-axis!

 This is where the "Discriminant" helps us ...

Discriminant

Do you see b2 − 4ac in the formula above? It is called the Discriminant, because it can "discriminate" between
the possible types of answer:

 when b2 − 4ac is positive, we get two Real solutions

 when it is zero we get just ONE real solution (both answers are the same)

 when it is negative we get a pair of Complex solutions

Complex solutions? Let's talk about them after we see how to use the formula.

Using the Quadratic Formula

Just put the values of a, b and c into the Quadratic Formula, and do the calculations.

The discriminant tells us what kind of roots a quadratic equation has.

Value of b2 − 4ac Solutions

Positive Both real

Zero One real (with duplicity), one complex

Negative Both complex

Sum of roots: -b/a

Product of roots: c/a

Special quadratic functions (for the following, C is a constant):

f (x) = x2 − C2 = (x − c)(x + c)

f (x) = x2 ± 2Cx + C2 = (x ± C)2

Factoring a quadratic using the AC method

121
This document is not for sale.

We will be using the following equation as an example:

8x2 − 6x − 5

a = 8, b = − 6, c = − 5

① Identify two factors of a c that add up to b.

ac = 8(−5) = − 40

factors = {−10,4}, since − 10 + 4 = − 6 = b

② Divide ! by the factors identified in ①.

8/-10; 8/4

③ Simplify both fractions.

8/-10 = 4/-5; 8/4 = 2/1

④ Use the simplified fractions as coefficients of the factors of the quadratic equation.

4/-5 = (4x-5); 2/1=(2x+1)

⑤ Check that the factors produce the original quadratic equation.

(4x-5)(2x+1)=8x2-6x-5

Example: Solve 5x2 + 6x + 1 = 0

Coefficients are: a = 5, b = 6, c = 1

Quadratic Formula: x = −b ± √(b2 − 4ac)/2a

Put in a, b and c: x = −6 ± √(62 − 4×5×1)/2×5

Solve: x = −6 ± √(36 – 20)/10

x = −6 ± √(16)/10

x = −6 ± 4/10

x = −0.2 or −1

Answer: x = −0.2 or x = −1

And we see them on this graph.

122
This document is not for sale.

Check -0.2: 5×(−0.2)2 + 6×(−0.2) + 1

= 5×(0.04) + 6×(−0.2) + 1

= 0.2 − 1.2 + 1

=0

Check -1: 5×(−1)2 + 6×(−1) + 1

= 5×(1) + 6×(−1) + 1

=5−6+1

=0

When the Discriminant (the value b2 − 4ac) is negative we get a pair of Complex solutions ... what does that
mean?

It means our answer will include Imaginary Numbers. Wow!

Example: Solve 5x2 + 2x + 1 = 0

Coefficients are: a=5, b=2, c=1

Note that the Discriminant is negative: b2 − 4ac = 22 − 4×5×1

= −16

Use the Quadratic Formula: x = −2 ± √(−16)/10

√(−16) = 4i

(where i is the imaginary number √−1)

So: x = −2 ± 4i/10

Answer: x = −0.2 ± 0.4i

123
This document is not for sale.

The graph does not cross the x-axis. That is why we ended up with complex numbers.
In some ways it is easier: we don't need more calculation, just leave it as −0.2 ± 0.4i.

Example: Solve x2 − 4x + 6.25 = 0

Coefficients are: a=1, b=−4, c=6.25

Note that the Discriminant is negative: b2 − 4ac = (−4)2 − 4×1×6.25

= −9

Use the Quadratic Formula: x = −(−4) ± √(−9)/2

√(−9) = 3i

(where i is the imaginary number √−1)

So: x = 4 ± 3i/2

Answer: x = 2 ± 1.5i

The graph does not cross the x-axis. That is why we ended up with complex numbers.

BUT an upside-down mirror image of our equation does cross the x-axis at 2 ± 1.5 (note: missing the i).

Just an interesting fact for you!

The simplest Quadratic Equation is: f(x) = x2

And its graph is simple too:

124
This document is not for sale.

This is the curve f(x) = x2

It is a parabola.

Now let us see what happens when we introduce the "a" value:

f(x) = ax2

 Larger values of a squash the curve inwards

 Smaller values of a expand it outwards

 And negative values of a flip it upside down

Before graphing we rearrange the equation, from this: f(x) = ax2 + bx + c

To this: f(x) = a(x-h)2 + k

Where:

h = −b/2a

k = f(h)

In other words, calculate h (= −b/2a), then find k by calculating the whole equation for x=h

The wonderful thing about this new form is that h and k show us the very lowest (or very highest) point, called
the vertex:

125
This document is not for sale.

And also the curve is symmetrical (mirror image) about the axis that passes through x=h, making it easy to
graph

So...

h shows us how far left (or right) the curve has been shifted from x=0

k shows us how far up (or down) the curve has been shifted from y=0

Lets see an example of how to do this:

Example: Plot f(x) = 2x2 − 12x + 16

First, let's note down:

a = 2,

b = −12, and

c = 16

Now, what do we know?

 a is positive, so it is an "upwards" graph ("U" shaped)

 a is 2, so it is a little "squashed" compared to the x2 graph

Next, let's calculate h:

h = −b/2a = −(−12)/(2x2) = 3

And next we can calculate k (using h=3):

k = f(3) = 2(3)2 − 12·3 + 16 = 18−36+16 = −2

So now we can plot the graph (with real understanding!):

We also know: the vertex is (3,−2), and the axis is x=3

126
This document is not for sale.

What if we have a graph, and want to find an equation?

Example: you have just plotted some interesting data, and it looks Quadratic:

Just knowing those two points we can come up with an equation.

Firstly, we know h and k (at the vertex):

(h, k) = (1, 1)

So let's put that into this form of the equation:

f(x) = a(x-h)2 + k

f(x) = a(x−1)2 + 1

Then we calculate "a":

We know the point (0, 1.5) so: f(0) = 1.5

And a(x−1)2 + 1 at x=0 is: f(0) = a(0−1)2 + 1

They are both f(0) so make them equal: a(0−1)2 + 1 = 1.5

Simplify: a + 1 = 1.5

a = 0.5

And so here is the resulting Quadratic Equation:

f(x) = 0.5(x−1)2 + 1

Note: This may not be the correct equation for the data, but it’s a good model and the best we can come up with.

Quadratic Inequality

But when we solve inequalities we try to find interval(s), such as the ones marked ">0" or "<0"

127
This document is not for sale.

So this is what we do:

 find the "=0" points

 in between the "=0" points, are intervals that are either

o greater than zero (>0), or

o less than zero (<0)

 then pick a test value to find out which it is (>0 or <0)

Here is an example:

Example: x2 − x − 6 < 0

x2 − x − 6 has these simple factors (because I wanted to make it easy!):

(x+2)(x−3) < 0

Firstly, let us find where it is equal to zero:

(x+2)(x−3) = 0

It is equal to zero when x = −2 or x = +3

because when x = −2, then (x+2) is zero

or when x = +3, then (x−3) is zero

So between −2 and +3, the function will either be

 always greater than zero, or

128
This document is not for sale.

 always less than zero

We don't know which ... yet!

Let's pick a value in-between and test it:

At x=0: x2 − x − 6

= 0−0−6

= −6

So between −2 and +3, the function is less than zero.

And that is the region we want, so...

x2 − x − 6 < 0 in the interval (−2, 3)

Note: x2 − x − 6 > 0 on the interval (−∞,−2) and (3, +∞)

And here is the plot of x2 − x − 6:

 The equation equals zero at −2 and 3

 The inequality "<0" is true between −2 and 3.

129
This document is not for sale.

What If It Doesn't Go Through Zero?

Here is the plot of x2 − x + 1

There are no "=0" points!

But that makes things easier!

Because the line does not cross through y=0, it must be either:

 always > 0, or

 always < 0

So all we have to do is test one value (say x=0) to see if it is above or below.

Trigonometry
Trigonometry is the branch of mathematics that deals with the relationship between the sides and angles of
triangles.

Pythagorean Theorem

The Pythagorean theorem states that in a right triangle, a2+b2=c2, where a and b are the length of the legs, and c
is the length of the hypotenuse.

130
This document is not for sale.

Example

C2=62+42
C2=36+16
C2=52
C=√52
C≈7.2

There are a couple of special types of right triangles, like the 45°-45° right triangles and the 30°-60° right
triangle.

131
This document is not for sale.

Because of their angles it is easier to find the hypotenuse or the legs in these right triangles than in all other
right triangles.

In a 45°-45° right triangle we only need to multiply one leg by √2 to get the length of the hypotenuse.

Example

We multiply the length of the leg which is 7 inches by √2 to get the length of the hypotenuse.

7⋅√2≈9.9

In a 30°-60° right triangle we can find the length of the leg that is opposite the 30° angle by using this formula:

a=1/2⋅c

Example

To find a, we use the formula above.

a=1/2⋅14
a=7

132
This document is not for sale.

Terminologies and Formulae

Domain: The set of all possible inputs of a function.

Function: An operation that assigns a correspondence from elements of one set to elements of another set.

Period: The repeating interval of a periodic function; the period of a function is a real number.

Periodic Function: A function that repeats itself in regular intervals; it follows this equation: f (x + c) = f (x),
where c is a constant.

Range: The set of all possible outputs of a function.

Reference Angle: The positive acute angle formed between the terminal side of an angle and the x-axis.

Rule: The part of a function that dictates the exact correspondence between the elements of one set and the
elements of another set.

Trigonometric Functions: There are six trigonometric functions: Sine, cosine, tangent, cosecant, secant, and
cotangent.

Unit Circle: The circle whose center is at the origin and whose radius is one.

Cosecant: Given a point P(x, y) on the terminal side of an angle θ in standard position, distance d from the
origin, cosecant(θ) = csc(θ) =

Cosine: Given a point P(x, y) on the terminal side of an angle θ in standard position, distance d from the origin,
cosine(θ) = cos(θ) = .

133
This document is not for sale.

Cotangent: Given a point P(x, y) on the terminal side of an angle θ in standard position, cotangent(θ) = cot(θ) =
.

Secant Given a point P(x, y) on the terminal side of an angle θ in standard position, distance d from the origin,
secant(θ) = sec(θ) = .

134
This document is not for sale.

Sine: Given a point P(x, y) on the terminal side of an angle θ in standard position, distance d from the origin,
sine(θ) = sin(θ) = .

Tangent: Given a point P(x, y) on the terminal side of an angle θ in standard position, tangent(θ) = tan(θ) = .

Right Angled Triangle

The triangle of most interest is the right-angled triangle. The right angle is shown by the little box in the corner:

135
This document is not for sale.

Another angle is often labeled θ, and the three sides are then called:

 Adjacent: adjacent (next to) the angle θ

 Opposite: opposite the angle θ

 and the longest side is the Hypotenuse

Sine, Cosine and Tangent

The main functions in trigonometry are Sine, Cosine and Tangent. They are simply one side of a right-angled
triangle divided by another.

For any angle "θ":

(Sine, Cosine and Tangent are often abbreviated to sin, cos and tan.)

Example: What is the sine of 35°?

Using this triangle (lengths are only to one decimal place):

sin(35°) = Opposite/Hypotenuse = 2.8/4.9 = 0.57...

Example: How Tall is The Tree?

136
This document is not for sale.

Start with: sin(45°) = Opposite/Hypotenuse

We know: 0.7071... = Opposite/20

Swap sides: Opposite/20 = 0.7071...

Multiply both sides by 20: Opposite = 0.7071... × 20

Calculate: Opposite = 14.14 (to 2 decimals)

The tree is 14.14m tall.

137
This document is not for sale.

Unit Circle

It is a circle with a radius of 1 with its center at 0. Because the radius is 1, we can directly measure sine, cosine
and tangent. This is used to calculate the cosine, sine, and tangent of any angle between 0 and 360 degrees or
between 0 and 2 radians.

Here we see the sine function being made by the unit circle:

138
This document is not for sale.

Angles can be in Degrees or Radians. Here are some examples:

Angle Degrees Radians

Right angle 90° π/2

Straight Angle 180° π

Full Rotation 360° 2π

Because the angle is rotating around and around the circle the Sine, Cosine and Tangent functions repeat once
every full rotation (see Amplitude, Period, Phase Shift and Frequency).

When we want to calculate the function for an angle larger than a full rotation of 360° (2π radians) we subtract
as many full rotations as needed to bring it back below 360° (2π radians):

Example: what is the cosine of 370°?

370° is greater than 360° so let us subtract 360°

370° − 360° = 10°

cos(370°) = cos(10°) = 0.985 (to 3 decimal places)

And when the angle is less than zero, just add full rotations.

Example: what is the sine of −3 radians?

−3 is less than 0 so let us add 2π radians

−3 + 2π = −3 + 6.283... = 3.283... radians

sin(−3) = sin(3.283...) = −0.141 (to 3 decimal places)

Trigonometry is also useful for general triangles, not just right-angled ones .

It helps us in Solving Triangles. "Solving" means finding missing sides and angles.

Example: Find the Missing Angle "C"

139
This document is not for sale.

Angle C can be found using the fact that angles of a triangle add to 180°:

So C = 180° − 76° − 34° = 70°

We can also find missing side lengths. The general rule is:

When we know any 3 of the sides or angles we can find the other 3 (except for the three angles case)

Other Functions (Cotangent, Secant, Cosecant)

Similar to Sine, Cosine and Tangent, there are three other trigonometric functions which are made by dividing
one side by another:

Cosecant Function: csc(θ) = Hypotenuse / Opposite

Secant Function: sec(θ) = Hypotenuse / Adjacent

Cotangent Function: cot(θ) = Adjacent / Opposite

Therefore,

sin(θ) = 1/csc(θ)

cos(θ) = 1/sec(θ)

tan(θ) = 1/cot(θ)

and

140
This document is not for sale.

csc(θ) = 1/sin(θ)

sec(θ) = 1/cos(θ)

cot(θ) = 1/tan(θ)

and

cot(θ) = cos(θ)/sin(θ)

Use the magic hexagon to remember the operations of these functions.

Patterns and Sequences


Numbers can take up different patterns and sequences. Numbers that follow a certain consistent sequence is
called a pattern. A series is the sum of every term in a sequence.

Number Patterns

Arithmetic Sequence: Made by adding the same value each time. Has a common difference.

141
This document is not for sale.

Geometric Sequence: Made by multiplying the same value each time. Has a common ratio.

Harmonic Sequence: Made by taking the reciprocals of an arithmetic sequence.

Fibonacci Sequence: Made by adding the two numbers before the nth term together.

142
This document is not for sale.

Arithmetic Sequence

Arithmetic sequences follow the following formula

Arithmetic series follow the following formula

143
This document is not for sale.

Geometric Sequence

Geometric Sequences follow the following formula

Geometric Series follow the follow the following formula

144
This document is not for sale.

Harmonic Sequence

Harmonic Sequences follow the following formula

Fibonacci Sequence

145
This document is not for sale.

Fibonacci Sequences follow the following formula

Pascal’s Triangle
Pascal’s triangle can be used to figure out the coefficients when putting exponents on binomials, aka binomial
expansion.

Division of Polynomials
Long Division

The Long Division of Polynomials is a complex method of division.

146
This document is not for sale.

 Divide the first term of the numerator by the first term of the denominator, and put that in the answer.

 Multiply the denominator by that answer, put that below the numerator

 Subtract to create a new polynomial

 Repeat, using the new polynomial

147
This document is not for sale.

Synthetic Division

Synthetic Division is a simpler alternative to Long Division.

Divide:

Step 1: To set up the problem, first, set the denominator equal to zero to find the number to put in the division
box. Next, make sure the numerator is written in descending order and if any terms are missing you must use a
zero to fill in the missing term, finally list only the coefficient in the division problem.

Step 2: Once the problem is set up correctly, bring the leading coefficient (first number) straight down.

Step 3: Multiply the number in the division box with the number you brought down and put the result in the
next column.

148
This document is not for sale.

Step 4: Add the two numbers together and write the result in the bottom of the row.

Step 5: Multiply the number in the division box with the number you brought down and put the result in the
next column.

Step 6: Add the two numbers together and write the result in the bottom of the row.

Step 7: Multiply the number in the division box with the number you brought down and put the result in the
next column.

Step 8: Add the two numbers together and write the result in the bottom of the row.

Step 9: Write the final answer. The final answer is made up of the numbers in the bottom row with the last
number being the remainder and the remainder must be written as a fraction. The variables or x’s start off one
power less than the original denominator and go down one with each term.

Divide:

149
This document is not for sale.

Step 1: To set up the problem, first, set the denominator equal


to zero to find the number to put in the division box. Next,
make sure the numerator is written in descending order and if
any terms are missing you must use a zero to fill in the missing
term, finally list only the coefficient in the division problem.

Step 2: Once the problem is set up correctly, bring the leading


coefficient (first number) straight down.

Step 3: Multiply the number in the division box with the


number you brought down and put the result in the next
column.

Step 4: Add the two numbers together and write the result in
the bottom of the row.

Step 5: Multiply the number in the division box with the


number you brought down and put the result in the next
column.

Step 6: Add the two numbers together and write the result in
the bottom of the row.

Step 7: Multiply the number in the division box with the


number you brought down and put the result in the next
column.

Step 8: Add the two numbers together and write the result in
the bottom of the row.

Step 9: Write the final answer. The final answer is made up of


the numbers in the bottom row with the last number being the
remainder and the remainder must be written as a fraction. The
variables or x’s start off one power less than the original
denominator and go down one with each term.

150
This document is not for sale.

151
This document is not for sale.

152
This document is not for sale.

Permutation and Combination


Permutation

Permutation is an arrangement of objects in which the order of said objects does matter. Permutation has two
types, one where repetition is allowed, and one where repetition isn’t.

 Permutations with Repetition - When a thing has n different types ... we have n choices each time!

For example: choosing 3 of those things, the permutations are: n × n × n (n multiplied 3 times)

More generally: choosing r of something that has n different types, the permutations are: n × n × ... (r times)

(In other words, there are n possibilities for the first choice, THEN there are n possibilities for the second
choice, and so on, multiplying each time.)

Which is easier to write down using an exponent of r: n × n × ... (r times) = nr

Example: in the lock above, there are 10 numbers to choose from (0,1,2,3,4,5,6,7,8,9) and we choose 3 of
them: 10 × 10 × ... (3 times) = 103 = 1,000 permutations

So, the formula is simply:

nr, where n is the number of things to choose from, and we choose r of them, repetition is allowed, and order
matters.

153
This document is not for sale.

 Permutations without Repetition - In this case, we have to reduce the number of available choices each
time.

Example: what order could 16 pool balls be in?

After choosing, say, number "14" we can't choose it again.

So, our first choice has 16 possibilities, and our next choice has 15 possibilities, then 14, 13, 12, 11, ... etc.
And the total permutations are: 16 × 15 × 14 × 13 × ... = 20,922,789,888,000

But maybe we don't want to choose them all, just 3 of them, and that is then: 16 × 15 × 14 = 3,360

In other words, there are 3,360 different ways that 3 pool balls could be arranged out of 16 balls. Without
repetition our choices get reduced each time.

But how do we write that mathematically? Answer: we use the "factorial function"

The factorial function (symbol: !) just means to multiply a series of descending natural numbers. Examples:

4! = 4 × 3 × 2 × 1 = 24

7! = 7 × 6 × 5 × 4 × 3 × 2 × 1 = 5,040

1! = 1

Note: it is generally agreed that 0! = 1. It may seem funny that multiplying no numbers together gets us 1,
but it helps simplify a lot of equations.

So, when we want to select all of the billiard balls the permutations are: 16! = 20,922,789,888,000

But when we want to select just 3 we don't want to multiply after 14. How do we do that? There is a neat
trick: we divide by 13!

16 × 15 × 14 × 13 × 12 ...13 × 12 ... = 16 × 15 × 14

That was neat. The 13 × 12 × ... etc gets "cancelled out", leaving only 16 × 15 × 14.

The formula is written:

n!/(n − r)!, where n is the number of things to choose from, and we choose r of them, no repetitions, order
matters.

Example Our "order of 3 out of 16 pool balls example" is:

16!/ (16-3)! = 16!/ 13! = 20,922,789,888,000/6,227,020,800 = 3,360

(which is just the same as: 16 × 15 × 14 = 3,360)

Example: How many ways can first and second place be awarded to 10 people?

154
This document is not for sale.

10!/(10-2)! = 10!/8! = 3,628,800/40,320 = 90

(which is just the same as: 10 × 9 = 90)

Notation: Instead of writing the whole formula, people use different notations such as these:

Example: P(10,2) = 90

155
This document is not for sale.

Combination

Permutation is an arrangement of objects in which the order of said objects does matter. Combinations also have
the same two types as Permutations.

 Combinations with Repetition – OK, now we can tackle this one ...

Let us say there are five flavors of icecream: banana, chocolate, lemon, strawberry and vanilla.

We can have three scoops. How many variations will there be?

Let's use letters for the flavors: {b, c, l, s, v}. Example selections include

{c, c, c} (3 scoops of chocolate)

{b, l, v} (one each of banana, lemon and vanilla)

{b, v, v} (one of banana, two of vanilla)

(And just to be clear: There are n=5 things to choose from, and we choose r=3 of them. Order does not
matter, and we can repeat!)

Now, I can't describe directly to you how to calculate this, but I can show you a special technique that lets
you work it out.

156
This document is not for sale.

Think about the ice cream being in boxes, we could say "move past the first box, then take 3 scoops, then
move along 3 more boxes to the end" and we will have 3 scoops of chocolate!

So it is like we are ordering a robot to get our ice cream, but it doesn't change anything, we still get what we
want.

We can write this down as (arrow means move, circle means scoop).

In fact the three examples above can be written like this:

{c, c, c} (3 scoops of chocolate):

{b, l, v} (one each of banana, lemon and vanilla):

{b, v, v} (one of banana, two of vanilla):

OK, so instead of worrying about different flavors, we have a simpler question: "how many different ways
can we arrange arrows and circles?"

Notice that there are always 3 circles (3 scoops of ice cream) and 4 arrows (we need to move 4 times to go
from the 1st to 5th container).

So (being general here) there are r + (n−1) positions, and we want to choose r of them to have circles.

This is like saying "we have r + (n−1) pool balls and want to choose r of them". In other words it is now like
the pool balls question, but with slightly changed numbers. And we can write it like this:

where n is the number of things to choose from, and we choose r of them repetition allowed, order doesn't
matter.

Interestingly, we can look at the arrows instead of the circles, and say "we have r + (n−1) positions and want
to choose (n−1) of them to have arrows", and the answer is the same:

So, what about our example, what is the answer?

(3+5−1)!/ 3!(5−1)!= 7!/ 3!×4! = 5040/6×24= 35

There are 35 ways of having 3 scoops from five flavors of ice cream.

157
This document is not for sale.

 Combinations without Repetition - This is how lotteries work. The numbers are drawn one at a time, and
if we have the lucky numbers (no matter what order) we win!

The easiest way to explain it is to: assume that the order does matter (ie permutations), then alter it so the
order does not matter.

Going back to our pool ball example, let's say we just want to know which 3 pool balls are chosen, not the
order.

We already know that 3 out of 16 gave us 3,360 permutations. But many of those are the same to us now,
because we don't care what order! For example, let us say balls 1, 2 and 3 are chosen. These are the
possibilities:

Order does matter Order doesn't matter

123 123

132

213

231

312

321

So, the permutations have 6 times as many possibilities.

In fact there is an easy way to work out how many ways "1 2 3" could be placed in order, and we have
already talked about it. The answer is:

3! = 3 × 2 × 1 = 6

(Another example: 4 things can be placed in 4! = 4 × 3 × 2 × 1 = 24 different ways, try it for yourself!)

So we adjust our permutations formula to reduce it by how many ways the objects could be in order
(because we aren't interested in their order any more):

That formula is so important it is often just written in big parentheses like this:

158
This document is not for sale.

where n is the number of things to choose from, and we choose r of them, no repetition, order doesn't
matter.

Notation

As well as the "big parentheses", people also use these notations:

Just remember the formula:

n!/r!(n-r)!

Example: Pool Balls (without order)

So, our pool ball example (now without order) is:

16!/3!(16−3)! = 16!/3! × 13!

= 20,922,789,888,0006 × 6,227,020,800

= 560

Or we could do it this way:

(16×15×14)/(3×2×1) = 3360/6 = 560

It is interesting to also note how this formula is nice and symmetrical:

n other words choosing 3 balls out of 16, or choosing 13 balls out of 16 have the same number of
combinations.

16!/3!(16−3)! = 16!/13!(16−13)! = 16!/3! × 13! = 560

Pascal's Triangle

We can also use Pascal's Triangle to find the values. Go down to row "n" (the top row is 0), and then along
"r" places and the value there is our answer. Here is an extract showing row 16:

1 14 91 364 ...

1 15 105 455 1365 ...

1 16 120 560 1820 4368 ...

159
This document is not for sale.

Measures of Position
A measure of position is a method by which the position that a particular data value has within a given data set
can be identified. As with other types of measures, there is more than one approach to defining such a measure.

These different measures of position are commonly used in statistics, and these are the following:

160
This document is not for sale.

161
This document is not for sale.

162
This document is not for sale.

163
This document is not for sale.

Decile

Deciles divide the entirety of the data set into 10 equal parts.

164
This document is not for sale.

Quartile

Quartiles divide the entirety of the data set into 4 equal parts.

165
This document is not for sale.

166
This document is not for sale.

Percentile

Percentiles divide the entirety of the data set into 100 equal parts.

167
This document is not for sale.

168
This document is not for sale.

Functions
In mathematics, a function is a binary relation over two sets that associates to every element of the first set
exactly one element of the second set. Typical examples are functions from integers to integers or from the real
numbers to real numbers.

169
This document is not for sale.

170
This document is not for sale.

171
This document is not for sale.

172
This document is not for sale.

173
This document is not for sale.

174
This document is not for sale.

175
This document is not for sale.

176
This document is not for sale.

177
This document is not for sale.

Revisiting Functions

The past functions we have tackled include linear and quadratic functions.

178
This document is not for sale.

Combination of Functions

In this section, we show how combinations of familiar functions together with some function operations can be
used to construct more complex functions. The rules involving the arithmetic combinations of functions are
quite natural. If f(x) = x − 1 and g(x) = x 2 − 4, we can form the sum, difference, product, and quotient of f and g
as follows.
a. Sum: (f + g)(x) = f(x) + g(x)
= (x − 1) + (x 2 − 4) = x 2 + x − 5
b. Difference: (f − g)(x) = f(x) − g(x)
= (x − 1) − (x 2 − 4) = x 2 + x + 3
c. Product: (f · g)(x) = f(x) · g(x)
= (x − 1) · (x 2 − 4) = x 3 − x 2 − 4x + 4
d. Quotient: ( f/g )(x) = f(x)/g(x) = x − 1 / x 2 − 4 = x – 1 / (x + 2)(x − 2) where: x ≠ ±2
The domain of an arithmetic combination of functions f and g consists of all real numbers that are common to
the domains of f and g. In the case of the quotient ( f/g )(x), there is the further restriction that g(x) ≠ 0.
Piece-Wise Functions

Piecewise functions are functions that are made up of various sub functions.

Example:

 when x is less than 2, it gives x2,

 when x is exactly 2 it gives 6

 when x is more than 2 and less than or equal to 6 it gives the line 10-x

It looks like this:

(a solid dot means "including",


an open dot means "not including")

179
This document is not for sale.

And this is how we write it:

The Domain (all the values that can go into the function) is all Real Numbers up to and including 6, which we
can write like this:

Dom(f) = (-∞, 6] (using Interval Notation)

Dom(f) = {x R | x ≤ 6} (using Set Builder Notation)

And here are some example values:

X Y

−4 16

−2 4

0 0

1 1

2 6

3 7

Example: Here is another piecewise function:

180
This document is not for sale.

which looks like:

What is h(−1)?

x is ≤ 1, so we use h(x) = 2, so h(−1) = 2

What is h(1)?

x is ≤ 1, so we use h(x) = 2, so h(1) = 2

What is h(4)?

x is > 1, so we use h(x) = x, so h(4) = 4

Inverse Functions

Inverse functions are functions that reverses another function.

This is easy -- it's just a list of steps. At this level, the problems are pretty simple.

Let's just do one, then I'll write out the list of steps for you.

Find the inverse of

STEP 1: Stick a "y" in for the "f(x)" guy:

STEP 2: Switch the x and y


(because every (x, y) has a (y, x) partner!):

181
This document is not for sale.

STEP 3: Solve for y:

STEP 4: Stick in the inverse notation,

182
This document is not for sale.

183
This document is not for sale.

Polynomial Functions

A polynomial function is a function which involves only non-negative integer powers or only positive integer
exponents of a variable in an equation like the quadratic equation, cubic equation, etc. For example, 2x+5 is a
polynomial which has exponent equal to 1.

y = Anxn + An−1xn−1 + An−2xn−2 +⋯+ A1x + A0

n = the degree of the function (highest power)

An = leading coefficient

Have at most n-1 turning points

Domain: all real numbers

If n is odd, the function is odd, i.e. f(−x) = − f (x), Range: all real numbers

If n is even, the function is even, i.e. f(−x) = f (x), Range: y > absolute minimum, or y < absolute maximum

184
This document is not for sale.

185
This document is not for sale.

Rational Functions

In mathematics, a rational function is any function which can be defined by a rational fraction, i.e. an algebraic
fraction such that both the numerator and the denominator are polynomials.

Y=f(x)/g(x), where f(x) and g(x) are polynomial functions


Asymptotes are lines that the function approaches as x or y goes to infinity
Vertical asymptotes are the lines x=A, where g(A)=0
Horizontal asymptotes: If the degrees of f(x) and g(x) are equal, then the horizontal asymptote is the quotient
of the leading coefficients. If the degree of f(x) > degree of g(x), then the horizontal asymptote is y=0. If the
degree of f(x) < degree of g(x), there is no horizontal asymptote
For most rational functions:
• Domain: all real numbers, except for horizontal asymptotes
• Range: all real numbers, except for vertical asymptotes

186
This document is not for sale.

Radical Functions

A radical function contains a radical expression with the independent variable (usually x) in the radicand.
Usually radical equations where the radical is a square root is called square root functions. The value of b tells
us where the domain of the radical function begins.

y=n(x-h)+k, where (h, k) represents the horizontal and vertical shifts, respectively
Domain:
• If n is odd, the domain is all real numbers
• If n is even, the domain is all x such that the radicand (x-h) ≥0.

187
This document is not for sale.

188
This document is not for sale.

Exponential Functions

Let’s start off this section with the definition of an exponential function.

If b is any number such that b>0 and b≠1 then an exponential function is a function in the form,

f(x)=bx

where b is called the base and x can be any real number.

Notice that the x is now in the exponent and the base is a fixed number. This is exactly the opposite from what
we’ve seen to this point. To this point the base has been the variable, x in most cases, and the exponent was a

189
This document is not for sale.

fixed number. However, despite these differences these functions evaluate in exactly the same way as those that
we are used to. We will see some examples of exponential functions shortly.

Before we get too far into this section we should address the restrictions on b. We avoid one and zero because in
this case the function would be,

f(x)=0x=0 and f(x)=1x=1

and these are constant functions and won’t have many of the same properties that general exponential functions
have.

Next, we avoid negative numbers so that we don’t get any complex values out of the function evaluation. For
instance, if we allowed b=−4 the function would be,

f(x)=(−4)x⇒f(1/2)=(−4)1/2=-4

and as you can see there are some function evaluations that will give complex numbers. We only want real
numbers to arise from function evaluation and so to make sure of this we require that b not be a negative
number.

y=ab(x-h)+k, where b>0, (h.k) is the vertex, h and k represent the horizontal and vertical shifts

Horizontal Asymptote: y=k Domain: all real numbers Range: If a>0, then all y>k; If a<0, then all y<k

Behavior of an exponential function

b>1 b<1 b=1

a > 0 increasing decreasing horizontal

a < 0 decreasing increasing horizontal

190
This document is not for sale.

191
This document is not for sale.

192
This document is not for sale.

Logarithmic Functions

In mathematics, the logarithm is the inverse function to exponentiation. That means the logarithm of a given
number x is the exponent to which another fixed number, the base b, must be raised, to produce that number x.

In its simplest form, a logarithm answers the question:

How many of one number do we multiply to get another number?

Example: How many 2s do we multiply to get 8?

Answer: 2 × 2 × 2 = 8, so we had to multiply 3 of the 2s to get 8

So the logarithm is 3

How to Write it

We write "the number of 2s we need to multiply to get 8 is 3" as:

log2(8) = 3

So these two things are the same:

193
This document is not for sale.

194
This document is not for sale.

Number How Many 10s Base-10 Logarithm


.. etc..

1000 1 × 10 × 10 × 10 log10(1000) =3
100 1 × 10 × 10 log10(100) =2
10 1 × 10 log10(10) =1
1 1 log10(1) =0
0.1 1 ÷ 10 log10(0.1) = −1
0.01 1 ÷ 10 ÷ 10 log10(0.01) = −2
0.001 1 ÷ 10 ÷ 10 ÷ 10 log10(0.001) = −3
.. etc..

A logarithm is the power to which a number must be raised in order to get some other number (see Section 3 of
this Math Review for more about exponents). For example, the base ten logarithm of 100 is 2, because ten
raised to the power of two is 100:

log 100 = 2

because

102 = 100

This is an example of a base-ten logarithm. We call it a base ten logarithm because ten is the number that is
raised to a power. The base unit is the number being raised to a power. There are logarithms using different base
units. If you wanted, you could use two as a base unit. For instance, the base two logarithm of eight is three,
because two raised to the power of three equals eight:

log2 8 = 3 because 23 = 8

In general, you write log followed by the base number as a subscript. The most common logarithms are base 10
logarithms and natural logarithms; they have special notations. A base ten log is written

log

and a base ten logarithmic equation is usually written in the form:

log a = r

195
This document is not for sale.

A natural logarithm is written

ln

and a natural logarithmic equation is usually written in the form:

ln a = r

So, when you see log by itself, it means base ten log. When you see ln, it means natural logarithm (we'll define
natural logarithms below). In this course only base ten and natural logarithms will be used.

The natural logarithm of a number is its logarithm to the base of the mathematical constant e, where e is an
irrational and transcendental number approximately equal to 2.718281828459. The natural logarithm of x is
generally written as ln x, loge x, or sometimes, if the base e is implicit, simply log x.

Logarithmic functions are the inverses of exponential functions. The inverse of the exponential function y =
ax is x = ay. The logarithmic function y = logax is defined to be equivalent to the exponential equation x = a y. y
= logax only under the following conditions: x = a y, a > 0, and a≠1.

y=a logb(x-h)+k, b>0, b≠1

Vertical Asymptote: Value of x such that the expression in the logarithm is zero

Domain: All values of x such that the expression in the logarithm is positive

b>1 b<1

a > 0 increasing decreasing

a < 0 decreasing increasing

196
This document is not for sale.

Trigonometric Functions

The trigonometric functions include the following 6 functions: sine, cosine, tangent, cotangent, secant, and
cosecant.

197
This document is not for sale.

y=Asin(Bx-C)+D or y=Acos(Bx-C)+D

 Amplitude: |A|

 Period: 2/B

 Horizontal Shift: C/B

 Vertical Shift: D

 Domain: All real numbers

 Range: (-A+D)≤x≤(A+D)

y=Atan(Bx-C)+D

 Amplitude: |A|

 Period: /B

 Horizontal Shift: C/B

 Vertical Shift: D

 Domain: All real numbers – Vertical Asymptotes

 Range: (-A+D)≤x≤(A+D)

198
This document is not for sale.

Composite Functions

In mathematics, function composition is an operation that takes two functions f and g and produces a function h
such that h(x) = f(g(x)). In this operation, the function g is applied to the result of applying the function f to x.

Domain: {x|x ∈ R, g(x) ∈ domain of f(x)}

199
This document is not for sale.

"Function Composition" is applying one function to the results of another:

f()  g()

The result of f() is sent through g()

It is written: (g º f)(x)

Which means: g(f(x))

Business Mathematics
Mathematics is widely utilized in the field of Business.

Terminologies

Interest (I): Payment from a borrower or deposit-taking financial institution to a lender or depositor of an
amount above repayment of the principal sum, at a particular rate.

Annuities: A series of payments made at equal intervals.

200
This document is not for sale.

Bonds & Stocks: Certificates that are sold to raise money for starting a new company or for expanding an
existing company.

Loans: The lending of money by one or more individuals, organizations, or other entities to other individuals,
organizations etc.

Principal (P): The original sum of money borrowed.

Rate (r): The amount of interest in percent on a yearly basis.

Time (t): Term of the loan. The period when the borrowed money is repaid and expressed in years.

Final Amount (A(t)): Also known as maturity/future/accumulated value. The total amount of money to be paid
after the interest being added.

Simple Interest

Simple Interest is the Type of interest that is most commonly calculated on short term loans, typically computed
on a yearly basis.

The following is the formula for the Final Amount

Ordinary Interest: Interest that is based on a 360 days a year and 30 days a month basis.

Exact Interest: Interest that is based on a 365 days a year basis.

201
This document is not for sale.

Interest Between Two Specific Dates

202
This document is not for sale.

List of Formulas
Simple Interest
1. I = Prt
2. I = A(t) − P
3. A(t) = P + I
4. A(t) = P(1 + rt)
5. P = I/rt

203
This document is not for sale.

6. P = A(t)/(1+rt)
7. r = I/Pt
8. t = I/Pr
Simple Interest by No. of Months
1. I = Pr(months/12)
Simple Interest by No. of Days
1. I o = Pr(D/360)
2. I e = Pr(D/365)
Simple Interest between Two Dates
1. I o at Actual Time
• I o = Pr(Actual No. of Days/360)
2. I o at Approximate Time
• I o = Pr(Approx. No. of Days/360)
3. I e at Actual Time
• I e = Pr(Actual No. of Days/365)
4. I e at Approximate Time
• I e = Pr (Approx. No. of Days/365)
Compound Interest

Compound interest is the addition of interest to the principal sum of a loan or deposit. It is the result of
reinvesting interest, rather than paying it out, so that interest in the next period is then earned on the principal
sum plus previously accumulated interest.

204
This document is not for sale.

205
This document is not for sale.

206
This document is not for sale.

Logic and the Truth Table


Logic is the study of principles and methods on the basis of sound argument and reasoning. We study whether
the process by which a conclusion drawn from asset of initial assumption is correct. Logic utilizes the concept
of proposition.

A Proposition is a declarative sentence that can be classified as true or false, with that classification being
referred to as its truth value.

207
This document is not for sale.

Propositions can either be Simple or Compound.

 Simple Proposition – a proposition that conveys one thought with no connecting words.

 Compound Proposition – a proposition that contains two or more simple propositions that are put together
using connective words.

The following are examples of logical connectives.

Propositional Logic, also known as sentential logic or statement logic, is the branch of logic that studies ways of
joining and/or modifying entire proposition, statement or sentences and how they interact with each other.

Boolean Logic is A type of computer science originally developed by mathematician George Boole in the mid –
1800s.

It is a mathematical function that maps argument to a value – true and false.

Truth Table: A handy little logical device that shows up not only in mathematics but also in Computer Science
and Philosophy, making it an awesome interdisciplinary tool. The notation may vary depending on what
discipline you’re working in, but the basic concepts are the same.

208
This document is not for sale.

Conjunction (∧): And

Disjunction (∨): Either ... or / Or

Negation (¬): Not / It is not

Conditional (→): If ... then

Biconditional (↔): If and only if / Iff

209
This document is not for sale.

210
This document is not for sale.

Mathematical Induction
Mathematical induction is a technique for proving a statement -- a theorem, or a formula -- that is asserted about
every natural number.

1 + 2 + 3 + . . . + n = ½n(n + 1).

This asserts that the sum of consecutive numbers from 1 to n is given by the formula on the right. We want to
prove that this will be true for n = 1, n = 2, n = 3, and so on. Now we can test the formula for any given
number, say n = 3:

1 + 2 + 3 = ½· 3· 4 = 6

-- which is true. It is also true for n = 4:

1 + 2 + 3 + 4 = ½· 4· 5 = 10.

But how are we to prove this rule for every value of n?

The method of proof is the following. We show that if the statement -- the rule -- is true for any specific number
k (e.g. 104), then it will also be true for its successor, k + 1 (e.g. 105). We then show that the statement will be
true for 1. It then follows that the statement will be true for 2. Therefore it will be true for 3. It will be true for
any natural number we name.

This is called the principle of mathematical induction.

If

1) when a statement is true for a natural number n = k, then it will also be true for its successor, n = k + 1;

and

2) the statement is true for n = 1; then the statement will be true for every natural number n.

To prove a statement by induction, we must prove parts 1) and 2) above.

The hypothesis of Step 1) -- "The statement is true for n = k" -- is called the induction assumption, or the
induction hypothesis. It is what we assume when we prove a theorem by induction.

Statistics
Statistics is the Branch of Mathematics that deals with the collection, analysis, and interpretation of data,
specifically, numerical data.

Descriptive Statistics: Statistics that describes a collection of information.

211
This document is not for sale.

Inferential Statistics: Statistics that not only describes, but also makes inferences about the data.

Population: Total set of observations that can be made.

Sample: Information from a fraction of the population.

Parameter: Numerical characteristic of a population.

Random Variable: Variables whose outcomes depend on a random phenomenon.

Discrete RV: Can only take on whole numbers.

Continuous RV: Can take on any value along the interval, e.g. decimals.

Measures of Central Tendencies

Mode: Most commonly occurring value in a group of data.

1, 2, 3, 4, 4, 5, 5, 5, 6, 6, 7, 8, 9; the mode is 5, with a frequency of 3.

Median: The value in the middle of the distribution when arranged in a certain manner. If there are two value in
the middle, merge them.

1, 2, 3, 4, 4, 5, 5, 5, 6, 6, 7, 8, 9; the median is 5. 1, 2, 3, 4; the median is 2.5.

Mean: Also known as the average. The sum of all the values divided by the total number of values in a
distribution.

1, 2, 3, 6; the mean is 3 since 1+2+3+6=12/4=3.

Probability

Probability is a measure of how likely an event is to occur. They can be written as fractions from 0 to 1, as
decimals from 0 tot 1, or as percents from 0 to 100.

212
This document is not for sale.

Probability has different properties. They are typically expressed using set notation.

Addition Property:

213
This document is not for sale.

For Mutually Exclusive Events:

Complement Property:

Addition and multiplication

Given two events that are mutually exclusive, the sum of each event’s probability is equal to the probability of
either event happening.

• Example: What is the probability of getting either a 1 or a 6 when rolling a die?

214
This document is not for sale.

1/6 + 1/6 = 1/3

Given two independent events A and B, the product of both events’ probabilities is equal to the probabilities of
event A and B happening.

• Example: Given two dice, what is the probability of getting a 2 on the first die and a 4 on the second die?

1/6  1/6 = 1/36

Fundamental Principles of Counting

 Tree Diagram – A diagram used to show the total number of possible outcomes in a probability experiment.

 Table or Organized Listing – A systematic way of listing the outcome through use of columns and rows.

 Fundamental Counting Principle – The Fundamental Counting Principle uses multiplication of the number
of ways each event in an experiment can occur to find the number of possible outcomes in a sample space

215
This document is not for sale.

216
This document is not for sale.

Experiments and Events

An experiment is an activity that produces results, which are called outcomes.

The collection of all outputs is called a sample space, and a subset from this sample space is called an event.

Events can have a union, an intersection, and a complement, which work in the same way as sets.

217
This document is not for sale.

218
This document is not for sale.

219
This document is not for sale.

Additional info for the Fundamental counting principle:

Given two independent events A and B: If m = the total number of ways that event A can happen; and n = the
total number of ways thet event B can happen; then The total number of ways that events A and B can happen
together is mn.

Combinatories

The different combinatories are permutation and combination, let us revisit them.

The following formulae denote the total number of objects as n, and the number of selected objects as r.

Permutations

Selection where order matters.

nPr=n!/(n-r)!

Circular permutations

220
This document is not for sale.

A special kind of permutation that deals with arranging objects in a circle.

For example, in circular permutations, the set {a,b,c,d,e} is the same as {b,c,d,e,a}, because circles can be
rotated.

Pn=(n-r)!

Combinations

Selection where order does NOT matter.

nCr=n!/(n-r)!r!

Set Theory

Sets were previously tackled, and as newly established, they can be used in statistics and probability as well.

P(A ∪ B) = P(A) + P(B) − P(A ∩ B)

Mean, Variance, and Standard Deviation

221
This document is not for sale.

222
This document is not for sale.

Formulating Hypothesis

What is Hypothesis Testing?

A statistical hypothesis is an assumption about a population parameter. This assumption may or may not be
true. Hypothesis testing refers to the formal procedures used by statisticians to accept or reject statistical
hypotheses.

Statistical Hypotheses

The best way to determine whether a statistical hypothesis is true would be to examine the entire population.
Since that is often impractical, researchers typically examine a random sample from the population. If sample
data are not consistent with the statistical hypothesis, the hypothesis is rejected.

There are two types of statistical hypotheses.

223
This document is not for sale.

 Null hypothesis: The null hypothesis, denoted by Ho, is usually the hypothesis that sample observations
result purely from chance.

 Alternative hypothesis: The alternative hypothesis, denoted by H1 or Ha, is the hypothesis that sample
observations are influenced by some non-random cause.

For example, suppose we wanted to determine whether a coin was fair and balanced. A null hypothesis might be
that half the flips would result in Heads and half, in Tails. The alternative hypothesis might be that the number
of Heads and Tails would be very different. Symbolically, these hypotheses would be expressed as

Ho: P = 0.5

Ha: P ≠ 0.5

Suppose we flipped the coin 50 times, resulting in 40 Heads and 10 Tails. Given this result, we would be
inclined to reject the null hypothesis. We would conclude, based on the evidence, that the coin was probably not
fair and balanced.

Can We Accept the Null Hypothesis?

Some researchers say that a hypothesis test can have one of two outcomes: you accept the null hypothesis or
you reject the null hypothesis. Many statisticians, however, take issue with the notion of "accepting the null
hypothesis." Instead, they say: you reject the null hypothesis or you fail to reject the null hypothesis.

Why the distinction between "acceptance" and "failure to reject?" Acceptance implies that the null hypothesis is
true. Failure to reject implies that the data are not sufficiently persuasive for us to prefer the alternative
hypothesis over the null hypothesis.

Hypothesis Tests

Statisticians follow a formal process to determine whether to reject a null hypothesis, based on sample data.
This process, called hypothesis testing, consists of four steps.

State the hypotheses. This involves stating the null and alternative hypotheses. The hypotheses are stated in
such a way that they are mutually exclusive. That is, if one is true, the other must be false.

Formulate an analysis plan. The analysis plan describes how to use sample data to evaluate the null hypothesis.
The evaluation often focuses around a single test statistic.

Analyze sample data. Find the value of the test statistic (mean score, proportion, t statistic, z-score, etc.)
described in the analysis plan.

Interpret results. Apply the decision rule described in the analysis plan. If the value of the test statistic is
unlikely, based on the null hypothesis, reject the null hypothesis.

Decision Errors

Two types of errors can result from a hypothesis test.

224
This document is not for sale.

Type I error: A Type I error occurs when the researcher rejects a null hypothesis when it is true. The
probability of committing a Type I error is called the significance level. This probability is also called alpha,
and is often denoted by α.

Type II error: A Type II error occurs when the researcher fails to reject a null hypothesis that is false. The
probability of committing a Type II error is called Beta, and is often denoted by β. The probability of not
committing a Type II error is called the Power of the test.

Decision Rules

The analysis plan includes decision rules for rejecting the null hypothesis. In practice, statisticians describe
these decision rules in two ways - with reference to a P-value or with reference to a region of acceptance.

P-value. The strength of evidence in support of a null hypothesis is measured by the P-value. Suppose the test
statistic is equal to S. The P-value is the probability of observing a test statistic as extreme as S, assuming the
null hypothesis is true. If the P-value is less than the significance level, we reject the null hypothesis.

Region of acceptance. The region of acceptance is a range of values. If the test statistic falls within the region of
acceptance, the null hypothesis is not rejected. The region of acceptance is defined so that the chance of making
a Type I error is equal to the significance level.

The set of values outside the region of acceptance is called the region of rejection. If the test statistic falls within
the region of rejection, the null hypothesis is rejected. In such cases, we say that the hypothesis has been
rejected at the α level of significance.

These approaches are equivalent. Some statistics texts use the P-value approach; others use the region of
acceptance approach. On this website, we tend to use the region of acceptance approach.

One-Tailed and Two-Tailed Tests

A test of a statistical hypothesis, where the region of rejection is on only one side of the sampling distribution, is
called a one-tailed test. For example, suppose the null hypothesis states that the mean is less than or equal to 10.
The alternative hypothesis would be that the mean is greater than 10. The region of rejection would consist of a
range of numbers located on the right side of sampling distribution; that is, a set of numbers greater than 10.

A test of a statistical hypothesis, where the region of rejection is on both sides of the sampling distribution, is
called a two-tailed test. For example, suppose the null hypothesis states that the mean is equal to 10. The
alternative hypothesis would be that the mean is less than 10 or greater than 10. The region of rejection would
consist of a range of numbers located on both sides of sampling distribution; that is, the region of rejection
would consist partly of numbers that were less than 10 and partly of numbers that were greater than 10.

225
This document is not for sale.

226
This document is not for sale.

227
This document is not for sale.

Sigma Notation
THIS —Σ—is the Greek letter sigma. We use it to indicate a sum.
For example:

This means that we are to repeatedly add kak. The first time we write it, we put k = 1. That is indicated by
the lower index of the letter sigma. The next time, we put k = 2, then 3, and so on, until we come to the upper
index, which in this case is 4.
In other words, we are to repeatedly add kak, which we call the argument of the sum, or the summand, starting
with k = 1 and ending with k = 4.
k is called a dummy index because it does not actually affect the sum, and we could indicate that sum using any
letter we please; for example j:

228
This document is not for sale.

An Introduction to Calculus
Calculus is the mathematical study of continuous change.

Limits of a Function

A limit tells us the value that a function approaches as that function's inputs get closer and closer to some
number. The idea of a limit is the basis of all calculus.

We say that the limit of f(x) is L as x approaches a and write this as limx→af(x)=L provided we can make f(x) as
close to L as we want for all x sufficiently close to a, from both sides, without actually letting x be a.

229
This document is not for sale.

230
This document is not for sale.

231
This document is not for sale.

232
This document is not for sale.

233
This document is not for sale.

234
This document is not for sale.

235
This document is not for sale.

236
This document is not for sale.

237
This document is not for sale.

Continuity

In calculus, a function is continuous at x = a if - and only if - all three of the following conditions are met:

1. The function is defined at x = a; that is, f(a) equals a real number

2. The limit of the function as x approaches a exists

3. The limit of the function as x approaches a is equal to the function value at x = a

We can define continuous using Limits (it helps to read that page first):

A function f is continuous when, for every value c in its Domain:

238
This document is not for sale.

f(c) is defined, and

limx→c f(x) = f(c)

"the limit of f(x) as x approaches c equals f(c)"

The limit says:

"as x gets closer and closer to c then f(x) gets closer and closer to f(c)"

And we have to check from both directions:

as x approaches c (from left) then f(x) approaches f(c)

AND as x approaches c (from right) then f(x) approaches f(c)

If we get different values from left and right (a "jump"), then the limit does not exist! And remember this has to
be true for every value c in the domain.

Make sure that, for all x values:

f(x) is defined and the limit at x equals f(x)

239
This document is not for sale.

240
This document is not for sale.

241
This document is not for sale.

Differentiation
Differentiation is the process of finding the derivative of a function, formally known as Differential Calculus.

242
This document is not for sale.

Common Functions Function Derivative

Constant c 0

Line x 1

ax a

Square x2 2x

Square Root √x (½)x-½

Exponential ex ex

ax ln(a) ax

243
This document is not for sale.

Logarithms ln(x) 1/x

loga(x) 1 / (x ln(a))

Trigonometry (x is in radians) sin(x) cos(x)

cos(x) −sin(x)

tan(x) sec2(x)

Inverse Trigonometry sin-1(x) 1/√(1−x2)

cos-1(x) −1/√(1−x2)

tan-1(x) 1/(1+x2)

Rules Function Derivative

Multiplication by constant cf cf’

Power Rule xn nxn−1

Sum Rule f+g f’ + g’

Difference Rule f-g f’ − g’

Product Rule fg f g’ + f’ g

Quotient Rule f/g (f’ g − g’ f )/g2

Reciprocal Rule 1/f −f’/f2

Chain Rule
fºg (f’ º g) × g’
(as "Composition of Functions")

Chain Rule (using ’ ) f(g(x)) f’(g(x))g’(x)

Chain Rule (using d/dx ) Dy/dx = dy/du du/dx

244
This document is not for sale.

Example: what is the derivative of sin(x) ?

From the table above it is listed as being cos(x)

It can be written as:

(d/dx)sin(x) = cos(x) Or: sin(x)’ = cos(x)

Example: What is (d/dx)x3 ?

The question is asking "what is the derivative of x3 ?"

We can use the Power Rule, where n=3:

(d/dx)xn = nxn−1

(d/dx)x3 = 3x3−1 = 3x2

(In other words the derivative of x3 is 3x2)

Example: What is d/dx(1/x) ?

1/x is also x-1

We can use the Power Rule, where n = −1:

(d/dx)xn = nxn−1

(d/dx)x−1 = −1x−1−1

= −x−2

= −1/x2

Example: What is d/dx(5x−2)3 ?

The Chain Rule says:

the derivative of f(g(x)) = f’(g(x))g’(x)

(5x-2)3 is made up of g3 and 5x-2:

f(g) = g3

g(x) = 5x−2

The individual derivatives are:

f'(g) = 3g2 (by the Power Rule)

245
This document is not for sale.

g'(x) = 5

So: d/dx(5x−2)3 = 3g(x)2 × 5 = 15(5x−2)2

Differentiation can also utilize the symbol Delta () which denotes an increment, or a change in a variable
quantity. A change in one side of the equation produces an equal change in the other side of the equation. y
corresponds to x.

246
This document is not for sale.

247
This document is not for sale.

248
This document is not for sale.

249
This document is not for sale.

250
This document is not for sale.

251
This document is not for sale.

252
This document is not for sale.

253
This document is not for sale.

254
This document is not for sale.

255
This document is not for sale.

256
This document is not for sale.

257
This document is not for sale.

258
This document is not for sale.

259
This document is not for sale.

Maxima and Minima

Many applications of calculus require us to deduce facts about a function f from the information concerning its
derivatives. Since f ‘ (x) represents the slope of the curve y = f(x) at the point (x, f(x)), it tells us the direction in
which the curve proceeds at each point.

Increasing / Decreasing Test

1. If f(x) > 0 on an interval, then f is increasing on that interval.

2. If f(x) < 0 on an interval, then f is decreasing on that interval.

Example:

Find where the function f(x) = x3 – 12x + 1 is increasing and where it is decreasing.

Solution:

Step 1: Find the derivative of f

f ‘(x) = 3x2 – 12 = 3(x2 – 4) = 3(x –2) (x + 2)

Step 2: Set f ‘(x) = 0 to get the critical numbers

f ‘(x) = 3(x –2) (x + 2) = 0

x = 2, –2

Step 3: Set up intervals whose endpoints are the critical numbers and determine the sign of f ‘(x) for each of the
intervals. Use the increasing/decreasing test to determine whether f(x) is increasing or decreasing for each
interval.

Interval x–2 x+2 f ‘(x) f

x<–2 – – + Increasing on

–2 < x < 2 – + – Decreasing on (–2, 2)

x>2 + + + Increasing on

The First Derivative Test

Suppose that c is a critical number of a continuous function f.

1. If f ‘ changes from positive to negative at c, then f has a local maximum at c.

2. If f ‘ changes from negative to positive at c, then f has a local minimum at c.

260
This document is not for sale.

3. If f ‘ does not change sign at c (f ‘ is positive at both sides of c or f ‘ is negative on both sides), then f has no
local maximum or minimum at c.

Example:

Find the local maximum and minimum values of the function f(x) = x4 – 2x2 + 3

Solution:

Step 1: Find the derivative of f

f ‘(x) = 4x3 – 4x = 4x(x2 –1) = 4x(x –1)(x +1)

Step 2: Set f ‘(x) = 0 to get the critical numbers

f ‘(x) = 4x(x –1)(x +1) = 0

x = –1, 0, 1

Step 3: Set up intervals whose endpoints are the critical numbers and determine the sign of f ‘(x) for each of the
intervals.

Interval 4x x-1 x+1 f ‘(x)

x < –1 – – – –

–1 < x < 0 – – + +

0<x<1 + – + –

x>1 + + + +

Step 4: Use the first derivative test to find the local maximum and minimum values.

f ‘(x) goes from negative to positive at x = –1, the First Derivative Test tells us that there is a local minimum at
x = –1.

f (–1) = 2 is the local minimum value.

f ‘(x) goes from positive to negative at x = 0, the First Derivative Test tells us that there is a local maximum at x
= 0.

f (0) = 3 is the local maximum value.

f ‘(x) goes from negative to positive at x = 1, the First Derivative Test tells us that there is a local minimum at x
= 1.

f (1) = 2 is the local minimum value.

261
This document is not for sale.

The Second Derivative Test

We can also use the Second Derivative Test to determine maximum or minimum values.

Suppose f ‘’ is continuous near c,

1. If f ‘(c) = 0 and f ‘’(c) > 0, then f has a local minimum at c.

2. If f ‘(c) = 0 and f ‘’(c) < 0, then f has a local maximum at c.

Example:

Use the Second Derivative Test to find the local maximum and minimum values of the function f(x) = x 4 – 2x2
+3

Solution:

Step 1: Find the derivative of f

f ‘(x) = 4x3 – 4x = 4x(x2 –1) = 4x(x –1)(x +1)

Step 2: Set f ‘(x) = 0 to get the critical numbers

f ‘(x) = 4x(x –1)(x +1) = 0

x = –1, 0, 1

Step 3: Find the second derivative

f ‘’(x) = 12x2 – 4

Step 4: Evaluate f ‘’at the critical numbers

f ‘’(–1) = 8 > 0, so f (–1) = 2 is the local minimum value.

f ‘’(0) = – 4 < 0, so f (0) = 2 is the local maximum value.

f ‘’(1) = 8 > 0, so f (1) = 2 is the local minimum value.

262
This document is not for sale.

263
This document is not for sale.

264
This document is not for sale.

265
This document is not for sale.

Integration
Integration is the process of obtaining the integral of a function, which is the reverse of differentiation. In
mathematics, an integral assigns numbers to functions in a way that can describe displacement, area, volume,
and other concepts that arise by combining infinitesimal data.

266
This document is not for sale.

267
This document is not for sale.

268
This document is not for sale.

269
This document is not for sale.

270
This document is not for sale.

Integration by Substitution.

271
This document is not for sale.

272
This document is not for sale.

273
This document is not for sale.

274
This document is not for sale.

275
This document is not for sale.

276
This document is not for sale.

277
This document is not for sale.

278
This document is not for sale.

Integration of Fractions

279
This document is not for sale.

280
This document is not for sale.

281
This document is not for sale.

282
This document is not for sale.

283
This document is not for sale.

284
This document is not for sale.

285
This document is not for sale.

286
This document is not for sale.

Integration Involving Quadratic Equation and Algebraic Substitution.

287
This document is not for sale.

288
This document is not for sale.

289
This document is not for sale.

290
This document is not for sale.

291
This document is not for sale.

Formula Summary

292
This document is not for sale.

293
This document is not for sale.

294
This document is not for sale.

295
This document is not for sale.

296
This document is not for sale.

297
This document is not for sale.

298
This document is not for sale.

299
This document is not for sale.

300
This document is not for sale.

301
This document is not for sale.

302
This document is not for sale.

303
This document is not for sale.

304
This document is not for sale.

305
This document is not for sale.

306
This document is not for sale.

307
This document is not for sale.

308
This document is not for sale.

309
This document is not for sale.

310
This document is not for sale.

311
This document is not for sale.

312
This document is not for sale.

313
This document is not for sale.

314
This document is not for sale.

Appendix

315
This document is not for sale.

References
1. Bonos, J. Business Mathematics, and General Mathematics.
2. Balantac, L. Basic Calculus.
3. Dantis J, Aguba I, Fajardo B, Fajardo L, Yadao J. Condensed Upcat Reviewer.
4. https://www.mathsisfun.com/calculus/integration-introduction.html
5. https://www.mathsisfun.com/calculus/derivatives-introduction.html
6. https://www.mathsisfun.com/calculus/derivatives-rules.html
7. https://www.onlinemathlearning.com/derivative-test.html
8. http://hyperphysics.phy-astr.gsu.edu/hbase/Math/maxmin.html
9. https://www.mathsisfun.com/calculus/maxima-minima.html
10. https://themathpage.com/aPreCalc/sigma.htm#:~:text=THIS%20%E2%80%94%CE%A3%E2%80%94is%20the,it%20to%20indicate%20a%20s
um.&text=This%20means%20that%20we%20are%20to%20repeatedly%20add%20ka%20k.
11. https://www.mathsisfun.com/calculus/continuity.html
12. https://study.com/academy/lesson/continuity-in-calculus-definition-examples-problems.html
13. https://tutorial.math.lamar.edu/problems/calci/continuity.aspx
14. https://tutorial.math.lamar.edu/classes/calci/thelimit.aspx
15. https://www.onlinemathlearning.com/limits-calculus.html
16. https://www.mathsisfun.com/calculus/limits.html
17. https://brilliant.org/wiki/limits-of-functions/
18. https://stattrek.com/hypothesis-test/hypothesis-testing.aspx
19. https://www.abs.gov.au/websitedbs/a3121120.nsf/home/statistical+language+-
+measures+of+central+tendency#:~:text=There%20are%20three%20main%20measures,occurring%20value%20in%20a%20distribution.
20. https://www.stat.berkeley.edu/~stark/SticiGui/Text/gloss.htm
21. https://conjointly.com/kb/sampling-statistical-terms/
22. https://www.dummies.com/education/math/statistics/terminology-used-in-
statistics/#:~:text=Here's%20an%20overview%20of%20the,a%20subset%20of%20that%20group.
23. https://themathpage.com/aPreCalc/mathematical-induction.htm
24. https://www.mathsisfun.com/sets/functions-composition.html
25. http://www.mclph.umn.edu/mathrefresh/logs.html
26. https://www.mathsisfun.com/algebra/logarithms.html
27. https://tutorial.math.lamar.edu/classes/alg/expfunctions.aspx
28. https://mathinsight.org/exponential_function
29. https://www.mathplanet.com/education/algebra-1/radical-expressions/the-graph-of-a-radical-
function#:~:text=A%20radical%20function%20contains%20a,is%20called%20square%20root%20functions.&text=The%20value%20of%20b%
20tells,of%20the%20radical%20function%20begins.
30. https://www.coolmath.com/algebra/16-inverse-functions/05-how-to-find-the-inverse-of-a-function-01
31. https://www.mathsisfun.com/sets/function-inverse.html
32. https://www.mathsisfun.com/sets/functions-piecewise.html
33. http://www.sirbonos.000webhostapp.com/
34. https://stattrek.com/descriptive-statistics/measures-of-
position.aspx#:~:text=Statisticians%20often%20talk%20about%20the,aka%2C%20z%2Dscores).
35. http://rhealikesmath.blogspot.com/2017/03/i.html
36. https://www.slideshare.net/erlynmgeronimo/measures-of-position-47928476
37. https://slideplayer.com/slide/6537920/
38. http://www.milefoot.com/math/stat/desc-positions.htm
39. https://www.mathsisfun.com/combinatorics/combinations-permutations.html
40. http://www.mesacc.edu/~scotz47781/mat120/notes/divide_poly/synthetic/synthetic_division.html
41. https://www.purplemath.com/modules/synthdiv.htm
42. https://www.mathsisfun.com/algebra/polynomials-division-long.html
43. https://www.math10.com/en/algebra/probabilities/binomial-theorem/binomial-theorem.html
44. https://www.mathsisfun.com/definitions/number-pattern.html
45. https://www.mathsisfun.com/numberpatterns.html
46. https://www.mathsisfun.com/algebra/triangle-identities.html
47. https://www.mathsisfun.com/algebra/trigonometry.html
48. https://www.mathsisfun.com/algebra/trig-magic-hexagon.html
49. https://www.mathsisfun.com/algebra/trigonometric-identities.html
50. https://www.mathsisfun.com/algebra/trigonometry.html
51. https://www.sparknotes.com/math/trigonometry/trigonometricfunctions/terms/
52. https://www.mathplanet.com/education/pre-algebra/right-triangles-and-algebra/the-pythagorean-
theorem#:~:text=triangles%20and%20algebra%20%2F-

316
This document is not for sale.

,The%20Pythagorean%20Theorem,sides%20in%20a%20right%20triangle.&text=The%20two%20legs%20meet%20at,side%20opposite%20the
%20right%20angle.
53. https://www.mathsisfun.com/algebra/quadratic-equation-graphing.html
54. https://www.mathsisfun.com/algebra/factoring-quadratics.html
55. https://www.mathsisfun.com/algebra/inequality-quadratic-solving.html
56. https://www.mathsisfun.com/algebra/quadratic-equation.html
57. https://www.mathsisfun.com/algebra/graphing-linear-inequalities.html
58. https://www.mathsisfun.com/algebra/inequality-solving.html
59. https://www.mathsisfun.com/algebra/inequality.html
60. https://www.mathsisfun.com/algebra/line-parallel-perpendicular.html
61. https://www.mathsisfun.com/algebra/line-midpoint.html
62. https://www.mathsisfun.com/algebra/line-equation-2points.html
63. https://www.mathsisfun.com/algebra/linear-equations.html
64. https://www.cliffsnotes.com/study-guides/geometry/fundamental-ideas/postulates-and-theorems
65. https://www.dummies.com/education/math/geometry/definitions-and-theorems-of-parallel-lines/
66. https://www.eastauroraschools.org/cms/lib/NY19000901/Centricity/Domain/285/Geometry%20theorems%20and%20postulates.htm
67. https://www.math-only-math.com/theorems-on-solid-geometry.html
68. https://www.eastauroraschools.org/cms/lib/NY19000901/Centricity/Domain/285/Geometry%20theorems%20and%20postulates.htm
69. http://www.ouchihs.org/ourpages/auto/2013/7/26/52822673/Geo-PostulatesTheorems-List.pdf
70. https://www.dummies.com/education/math/geometry/definitions-and-theorems-of-parallel-lines/
71. https://www.cliffsnotes.com/study-guides/geometry/fundamental-ideas/postulates-and-theorems
72. http://pirate.shu.edu/~kahlnath/Top100.html
73. http://people.math.harvard.edu/~knill/graphgeometry/papers/fundamental.pdf
74. https://mathbitsnotebook.com/Algebra1/RealNumbers/RNProp.html
75. http://www.solano.edu/academic_success_center/forms/math/Basic%20Number%20Properties.pdf
76. https://nrich.maths.org/6178
77. https://www.mathsisfun.com/algebra/factoring.html
78. https://socratic.org/questions/what-is-the-formula-for-the-surface-area-of-a-triangular-prism
79. https://www.sparknotes.com/math/precalc/conicsections/section4/#:~:text=Every%20hyperbola%20has%20two%20asymptotes,%2D%20(x%20
%2D%20h).
80. https://www.ck12.org/book/ck-12-college-precalculus/section/11.6/
81. https://www.varsitytutors.com/precalculus-help/conic-sections/hyperbolas
82. https://www.mathsisfun.com/geometry/hyperbola.html
83. https://courses.lumenlearning.com/waymakercollegealgebra/chapter/equations-of-ellipses/#:~:text=y%E2%88%92k)%20.-
,A%20General%20Note%3A%20Standard%20Forms%20of%20the%20Equation%20of%20an,with%20Center%20(h%2C%20k)&text=the%20
coordinates%20of%20the%20foci%20are%20(h%C2%B1c%2Ck,%3D%20a%202%20%E2%88%92%20b%202%20.
84. https://www.softschools.com/math/pre_calculus/ellipse_standard_equation/
85. http://cda.mrs.umn.edu/~mcquarrb/teachingarchive/Precalculus/Lectures/Ellipses.pdf
86. https://www.analyzemath.com/high_school_math/grade_11/parabola_problems.html
87. https://www.softschools.com/math/pre_calculus/parabola_standard_equation/
88. http://cda.mrs.umn.edu/~mcquarrb/teachingarchive/M1012/Lectures/10.1Parabolas.pdf
89. https://www.dummies.com/education/math/algebra/convert-parabolic-equations-to-the-standard-form/
90. https://www.varsitytutors.com/precalculus-help/conic-sections/parabolas
91. https://philschatz.com/precalculus-book/contents/m49442.html
92. https://philschatz.com/precalculus-book/contents/m49440.html
93. https://courses.lumenlearning.com/boundless-algebra/chapter/graphs-of-quadratic-functions/#:~:text=Key%20Points-
,The%20graph%20of%20a%20quadratic%20function%20is%20a,shaped%20curve%20called%20a%20parabola.&text=The%20extreme%20po
int%20(%20maximum%20or,parabola%20crosses%20the%20x%2Daxis.
94. https://www.sparknotes.com/math/precalc/conicsections/section2/#:~:text=A%20parabola%20is%20the%20set,the%20vertex%20is%20the%20
axis.
95. https://math.info/Geometry/Parallelograms/
96. https://math.info/Geometry/Rhombus/
97. https://www.dummies.com/education/math/calculus/pre-calculus-unit-circle/
98. https://mypages.valdosta.edu/alazari/math1111/Circle.html#:~:text=EQUATION%20OF%20A%20CIRCLE&text=2)%20The%20general%20f
orm%20%3A%20x,%2C%20r%2C%20is%20always%20positive.
99. https://math.info/Geometry/Quadrilaterals/
100. https://math.info/Geometry/
101. https://math.info/Calculus/
102. https://math.info/Arithmetic/
103. https://math.info/Algebra/Complex_Conjugates/
104. https://math.info/Algebra/
105. https://math.info/Geometry/Congruent_Triangles/

317
This document is not for sale.

106. https://math.info/Geometry/Equilateral_Triangles/
107. https://math.info/Geometry/Isosceles_Triangles/
108. https://math.info/Geometry/Congruent_Line_Segments/
109. https://math.info/Geometry/Scalene_Triangles/
110. https://slideplayer.com/slide/5702180/
111. https://www.mathwarehouse.com/geometry/circle/tangents-secants-arcs-angles.php
112. http://www.algebralab.org/lessons/lesson.aspx?file=Geometry_CircleSecantTangent.xml
113. https://www.engineersedge.com/math/circular_segment_equation_and_calculator__13796.htm
114. https://math.info/Geometry/Arcs_Major_Minor/
115. https://math.info/Geometry/Triangles_By_Angle_Measurements/
116. https://math.info/Geometry/Triangles/
117. https://math.info/Geometry/Circle/
118. https://math.info/Geometry/Circles_Product_Secants/
119. https://math.info/Geometry/Circle_Chord_Secant_Tangent/
120. https://math.info/Geometry/Circles_Product_Chords/
121. https://math.info/Geometry/Circle_Chord_Secant_Tangent/#:~:text=A%20line%20intersecting%20a%20circle,as%20the%20point%20of%20ta
ngency.
122. https://mathbitsnotebook.com/Geometry/Circles/CRSegmentRules.html
123. https://www.ck12.org/geometry/parts-of-circles/lesson/Parts-of-Circles-BSC-GEOM/
124. https://www.splashlearn.com/math-vocabulary/measurements/perimeter-of-a-triangle
125. https://mathbitsnotebook.com/Geometry/ParallelPerp/PPangles.html#:~:text=If%20two%20parallel%20lines%20are,alternate%20interior%20an
gles%20are%20congruent.&text=If%20two%20lines%20are%20cut,alternating%20sides%22%20of%20the%20transversal.
126. https://www.mathopenref.com/squarediagonals.html
127. https://www.wikihow.com/Find-the-Measurement-of-the-Diagonal-Inside-a-
Rectangle#:~:text=Square%20the%20length%20and%20width,multiply%20the%20number%20by%20itself.&text=%2C%20which%20is%20th
e%20hypotenuse%20of,the%20diagonal%20of%20the%20rectangle.&text=So%2C%20the%20diagonal%20of%20a,4%20cm%20is%205%20c
m.
128. https://www.hackmath.net/en/word-math-problems/diagonal
129. https://www.mathsisfun.com/geometry/polygons-diagonals.html
130. https://www.mathsisfun.com/definitions/diagonal.html#:~:text=A%20line%20segment%20that%20goes,edge%2C%20we%20get%20a%20diag
onal.
131. https://www.varsitytutors.com/precalculus-help/conic-sections/ellipses
132. https://www.varsitytutors.com/hotmath/hotmath_help/topics/cartesian-plane
133. https://www.rapidtables.com/math/symbols/Geometry_Symbols.html
134. https://www.varsitytutors.com/aplusmath/homework-helper/geoterms
135. https://www.chilimath.com/lessons/advanced-algebra/simplifying-complex-fractions/
136. https://gcseguide.co.uk/maths/algebra/algebraic-
expression/#:~:text=An%20algebraic%20expression%20is%20a,are%20examples%20of%20algebraic%20expression.&text=A%20variable%20
is%20a%20letter%20used%20to%20represent%20an%20unknown%20value.
137. http://algebrarules.com/
138. https://www.analyzemath.com/algebra/rules_algebra.html
139. https://sciencing.com/ways-tell-something-function-
8602995.html#:~:text=It%20is%20relatively%20easy%20to,be%20one%20greater%20than%20x.
140. https://byjus.com/maths/polynomial-functions/
141. https://www.sscc.edu/home/jdavidso/Math/Catalog/Polynomials/Polynomials.html
142. https://www.mathway.com/Precalculus
143. https://www.mesacc.edu/~scotz47781/mat120/notes/radicals/simplify/simplifying.html
144. https://www.theproblemsite.com/ask/2018/02/radicals-inside-radicals
145. https://courses.lumenlearning.com/boundless-algebra/chapter/radicals/
146. https://www.math10.com/en/algebra/radical.html
147. https://www.rapidtables.com/math/number/exponent.html
148. https://www.geteasysolution.com/what-number-is-3-less-than-17
149. https://www.infoplease.com/math-science/mathematics/algebra/algebra-translating-words-into-math
150. https://www.brainfuse.com/jsp/alc/resource.jsp?s=gre&c=35532&cc=108822
151. http://www.brainfuse.com/jsp/alc/resource.jsp?s=compass&c=42654&cc=120446
152. https://www.mathsisfun.com/length-conversion.html
153. https://www.mathsisfun.com/numbers/scientific-notation.html
154. https://www.factmonster.com/math-science/mathematics/converting-fractions-decimals-and-percents
155. https://www.varsitytutors.com/hotmath/hotmath_help/topics/percents-and-decimals
156. https://www.varsitytutors.com/hotmath/hotmath_help/topics/operations-with-decimals
157. https://www.varsitytutors.com/hotmath/hotmath_help/topics/fraction-operations

318
This document is not for sale.

158. http://www.learnalberta.ca/content/memg/Division03/Subset%20of%20a%20Set/index.html#:~:text=Subset%20of%20a%20Set,%22is%20a%2
0subset%20of%22.
159. https://www.cs.sfu.ca/~ggbaker/zju/math/set-oper.html
160. https://www.britannica.com/science/set-theory/Operations-on-sets
161. https://www.mathsisfun.com/sets/set-builder-notation.html
162. https://www.mathsisfun.com/sets/sets-introduction.html

319
This document is not for sale.

Additional Information
(View PH Tutorial Academy notes for other additional info)

320

Das könnte Ihnen auch gefallen